Re: [obm-l] QUANTOS ALGARISMOS

2007-10-09 Por tôpico Igor Castro
Vamos lá...
N =16^25 = 2^100
logN = 100*log2
logN=30,103
Logo N = 10^(30,103)= (10^30)*(10^0,103)

Repare que o primeiro fator desta multiplicação tem 31 digitos e começa com
1.
O segundo fator é um número maior que 1 e menor que 10.

Logo na multiplicao teremos:
1...000 * X
onde X é um número de 1 algarimo entre 1 e 10(exclusive). Logo teremos como
resposta um número da forma:
X...000.

Como já vimos, são 30 zeros mais o algarismo da esquerda.
Teremos assim 31 dígitos.

[]'s

On 10/9/07, arkon [EMAIL PROTECTED] wrote:

  *ALGUÉM PODE RESOLVER ESTA, POR FAVOR:*

 **

 *Quantos algarismos têm o número 1625? (se necessário use log 2 = 0,30103)
 *

 * *

 **

 *DESDE JÁ AGRADEÇO*



Re: [obm-l] Exercicio de fisica

2007-06-24 Por tôpico Igor Castro

Acho que essa não é a lista apropriada para discutir esse tipo de problema..
Procure listas de física no google.
[]'s

On 6/23/07, Camilo Damiao [EMAIL PROTECTED] wrote:


Por Favor, gostaria q alguem me ajudasse a encontrar a resposta desse
exercicio...

Um satelite de 20kgf foi colocado numa orbita circular distante 2500km
da superficie da Terra. Nesta altitude, a aceleração da gravidade é de
5,1 m/s². Determinar a energia cinética do satelite, sabendo-se que a
sua velocidade orbital é de 24000Km/h.
R: 453X10^5 kgm

Muito Obrigado pela atençãode tds!

=
Instruções para entrar na lista, sair da lista e usar a lista em
http://www.mat.puc-rio.br/~nicolau/olimp/obm-l.html
=



[obm-l] Integral - conjunto limitado

2007-06-23 Por tôpico Igor Castro

Caros amigos da lista,
como eu provo que se f(x) =  Integral(de 0 a x) de (1+t^3)^(-1/2)dt ,
então a imagem de f é um confunto limitado?
[]´s!


Re: [obm-l] funcao continua

2007-05-08 Por tôpico Igor Castro

Seja g(x) = f(x) - x
Logo, g é contínua. Mas:
g(1) = f(1) -1 = 0 e g(0) = f(0) - 0 = 0.
***Repare que só ocorre igualdade se f(1)=1 ou f(0)=0.
Descartando a igualdade temos que g(1)*g(0)  0. Logo existe uma raiz de g
entre 0 e 1(o nome do teorema eh bozano se nao me engano).
Se existe uma raiz a de g então g(a)=f(a)-a=0 - f(a)=a
[]'s

On 5/8/07, Klaus Ferraz [EMAIL PROTECTED] wrote:


Seja f:[0,1] - [0,1] uma função contínua. Provar que existe c E [0,1] tal
que f(c)=c.
vlw.

__
Fale com seus amigos de graça com o novo Yahoo! Messenger
http://br.messenger.yahoo.com/



Re: [obm-l] combinatoria

2007-03-13 Por tôpico Igor Castro

Seja o quarteto: AABC onde A é um violinista, B é um violista e C é um
violoncelista.
tenho:
6 A's = (A1,A2,A3,A4,A5,A6)
5 B's = (B1,B2,B3,B4,B5)
2C's + 3 B's = (C1,C2,B1,B2,B3))

Primeiro Caso:
Escolhendo os 2 A's: (6,2)
Escolhendo um B que só é B= (2,1)
Escolhendo o C = (5,1)

Segundo Caso:
Escolhendo os 2 A's: (6,2)
Escolhendo um B que é B e C= (3,1)
Escolhendo o C = (4,1)

Como os casos são disjuntos, devemos somar: (15.2.5)  +  (15.3.4) = 150 +
180 = 330 quartetos.

Abraços,
Igor.


On 3/13/07, Marcus Aurélio [EMAIL PROTECTED] wrote:


Alguem sabe fazer essa questão?

Um quarteto de cordas e formado por dois violinistas, um violista e um
violoncelista.
Quantos quartetos de cordas podem ser formados se dispomos de seis
violinistas,
cinco violistas, dos quais tres tamb?em podem tocar violoncelo, e dois
violoncelistas?



=
Instruções para entrar na lista, sair da lista e usar a lista em
http://www.mat.puc-rio.br/~nicolau/olimp/obm-l.html
=



Re: [obm-l] Conjuntos - dúvida conceitual

2007-03-07 Por tôpico Igor Castro

Eles tem que ser disjuntos dois a dois.

On 3/7/07, Pedro Cardoso [EMAIL PROTECTED] wrote:


Olá amigos da lista,

estudando um pouco conjuntos fiquei com uma dúvida em relação ao conceito
de
CONJUNTOS DISJUNTOS.

Entendi que A e B são disjuntos se A(inter)B = vazio, mas, quando começo a
trabalhar com três ou mais conjuntos...

Para N conjuntos serem disjuntos basta que a interseção simultãnea deles
[quero dizer A(inter)B(inter)C...(inter)Z] = vazio? Ou é necessário também
que eles sejam disjuntos dois a dois?

Se bastar que eles sejam disjuntos n a n [A(inter)B(inter)C...(inter)Z =
vazio], não é sempre verdade que:

A,B,C...Z são disjuntos - n[A(união)B...(união)Z] = n(A)+n(B)+...+n(Z),
certo?

Ex.: A ={1,2,3}; B ={2,4,6}; C={1,3,5}
A(inter)B(inter)C = vazio, mas [A(união)B(união)C] = {1,2,3,4,5,6} (6
elementos)

e n(A)+n(B)+n(C) = 3+3+3 = 9.

Grato,

Pedro Lazéra Cardoso

_
Mande torpedos SMS do seu messenger para o celular dos seus amigos
http://mobile.msn.com/

=
Instruções para entrar na lista, sair da lista e usar a lista em
http://www.mat.puc-rio.br/~nicolau/olimp/obm-l.html
=



[obm-l] Anéis não UFD

2007-02-24 Por tôpico Igor Castro

Olá, alguém poderia me dizer um exemplo de anel não-UFD(Unique
FactorizationDomain )
que não seja Z[sqrt(5)] ? Em particular eu gostaria de saber uma
aplicação, dentro ou fora da matemática, desses anéis???
Abraços,
Igor Castro.


[obm-l] ObmU 2006 - Questao 5

2006-10-31 Por tôpico Igor Castro
Alguem poderia mostrar como fez essa questao
O link pra prova é: http://www.obm.org.br/provas/obm2006/2Fase_Nivelu_2006.pdf


Re: [obm-l] ObmU 2006 - Questao 5

2006-10-31 Por tôpico Igor Castro
Necessária, mas é suficiente?

On 10/31/06, Ronaldo Luiz Alonso [EMAIL PROTECTED] wrote:
Igor Castro wrote: Alguem poderia mostrar como fez essa questao O link pra prova é:
 http://www.obm.org.br/provas/obm2006/2Fase_Nivelu_2006.pdfEu começaria notando que 1/f(n)  1/n é uma condição necessária para aconvergência e
que f^(-1)/n^2  1/n também (pelo teste da comparação) Assim se a primeira série converge f(n)  n e se a segunda sérieconverge f^(-1)(n)/n  1 == f^(-1)(n)  n.==(aplicando f dos dois lados)
n  f(n)(porque f é crescente).Então se a primeira converge a segunda converge e vice versa.Será que eu esqueci alguma coisa???AbraçoRonaldo.=
Instruções para entrar na lista, sair da lista e usar a lista emhttp://www.mat.puc-rio.br/~nicolau/olimp/obm-l.html=



[obm-l] Já pode discutir a prova da 3 fase???

2006-10-30 Por tôpico Igor Castro
Já pode discutir a prova da 3 fase???



Re: [obm-l] Linear

2006-08-26 Por tôpico Igor Castro
Basta vc notar que se (a,b) pertence a H, a = (3/5)*b - 6/5
Ese vc multiplicar por um escalar t vc tem (ta,tb), que para pertencer a H deve satisfazer ta = (3/5)tb - 6/5. Mas pela eq acima:ta = (3/5)*tb - (6/5)t que eh diferente de (3/5)tb - 6/5 a nao ser por t=1. Logo H não é um espaço vetorial.


ps: eu acho que estudei com vc curso intelecto, ou estou confundindo?

Igor Castro
[]'s
On 8/26/06, Marcus Aurélio [EMAIL PROTECTED] wrote:
Alguem me ajude por favor nesta questão:Mostre queo conjunto H formado por todos os pontos do R^2 da forma (3s,
2+5s) não é um espaço vetorial, arguementado que ele não é fechado com relaçãoà multiplicação por um escalar.obrigado=
Instruções para entrar na lista, sair da lista e usar a lista emhttp://www.mat.puc-rio.br/~nicolau/olimp/obm-l.html=



[obm-l] Re: [obm-l] OBM Nível U, problemas 5 e 6 da segund a fase 2005.

2006-05-03 Por tôpico Igor Castro

Bem legal a segunda solucao pro problema6(shine)!!!

- Original Message - 
From: Carlos Yuzo Shine [EMAIL PROTECTED]

To: obm-l@mat.puc-rio.br
Sent: Tuesday, May 02, 2006 12:59 PM
Subject: [obm-l] OBM Nível U, problemas 5 e 6 da segunda fase 2005.



Oi gente,

Na época da prova o Gugu e eu discutimos os problemas
5 e 6 da prova. Aí vão as soluções. A solução da 5 é
do Gugu. A 6 tem uma solução do Gugu e outra minha,
baseada em idéias que li no Proofs From The Book e que
também apareceram na OPM de alguns anos atrás.

Espero que estejam legíveis.

[]'s
Shine


- Forwarded message from [EMAIL PROTECTED] -
Date: Mon, 24 Oct 2005 19:22:26 -0200
From: [EMAIL PROTECTED]
Reply-To: [EMAIL PROTECTED]
 Subject: Re: OBMU
  To: Yuri Lima [EMAIL PROTECTED]

   Oi Yuri,
   Vamos lá:
5) Escreva x^(-x)=e^(-x.ln(x))=soma(n=0 a

infinito)((-x.ln(x))^n/n!). É só provar agora que
Integral(0 a 1)((x.ln(x))^n dx)=(-1)^n.n!/(n+1)^(n+1),
o que eu deixo para você fazer (qualquer problema me
avise).

6) Tem que provar que, se P_r(x)=Binomial(x+r,r),

então um polinômio do tipo f(x)=Soma(j=1 a
m)c_j.P_r_j(x), onde r_1r_2...r_m não pode ter m
raízes naturais distintas, se não for identicamente
nulo.

Para isso, provaremos por indução que para um tal

polinômio f(x), se c_m0, não existem i_1i_2...i_m
naturais com (-1)^(m-k).f(i_k)=0 para todo k=m. Para
m=1 isso é óbvio, pois f(x)0 sempre. Para r=s,
P_s(x)=P_r(x).P_(s-r)(x+r)/Binomial(s,r), e logo,
dividindo tudo por P_r_1(x), podemos supor r_1=0. A
partir daí, a indução se faz aplicando o operador M
(de menos) a f(x), dado por Mf(x)=f(x+1)-f(x).

Temos, para r=0, MP_r(x)=0 e, para r=1,

MP_r(x)=P_(r-1)(x+1). Detalhes com você.


 Oi Gugu,

 Antes, algumas definições.

 Considere o reticulado Z^2. Defina caminho entre

dois pontos P e Q de Z^2 como uma seqüência de pontos
do reticulado, cada um igual ao anterior mais (0,-1)
ou (1,0), com o primeiro termo igual a P e o último
igual a Q. Defina sistema de caminhos sem interseção
ligando dois subconjuntos X a Y de Z^2, cada um com n
elementos, como um conjunto de n caminhos disjuntos,
cada um ligando um ponto de X e um ponto de Y.


 Afirmação. Se A = (a_{rs}) é a matriz do problema

6 da OBM (lembrando que sua entrada a_{rs} é {i_r +
j_s\choose i_r}), então det(A) é igual ao número de
sistemas de caminhos sem interseção ligando X =
{(0,i_1),(0,i_2),...,(0,i_n)} a Y =
{(j_1,0),(j_2,0),...,(j_n,0)}.


 Note que a partir desse resultado o problema 6

agora se torna imediato, já que não é difícil achar um
sistema de caminhos sem interseção ligando X a Y (faça
uma figura e veja!).


 Demonstração da afirmação: Pela definição de

determinante, det(A) é a soma de n! termos, cada um
igual a sgn(p)a_{1p(1)}...a_{np(n)}, sendo p uma
permutação de (1,2,...,n). Considerando que a_{rs} =
{i_r + j_s\choose i_r}, esse termo sem o sinal é igual
ao número de maneiras de n caminhos ligarem os pares
de pontos (0,i_k) a (j_{p(k)},0), intersectando ou
não. Em particular, todos os nossos sistemas de
caminhos sem interseção estão sendo contados quando p
é a identidade (não é difícil provar que se p não é a
identidade então dois caminhos se intersectam; é só
fazer uma figura e usar continuidade). Então os
sistemas de caminhos sem interseção aparecem com o
sinal positivo no determinante.


 Os sistemas de caminhos com alguma interseção se

anulam no determinante: considere a interseção que
está mais à esquerda (ou seja, com abscissa mínima);
caso haja mais de uma, tome a que está mais para baixo
(com ordenada mínima). Suponha que a interseção seja
entre os caminhos ligando os pares (0,i_l), (j_m,0) e
(0,i_p), (j_q,0). Esse sistema de caminhos está sendo
contado numa parcela do determinante com dois fatores
iguais a a_{lm} e a_{pq}. Acontece que podemos obter
uma sistema de caminhos com as mesmas arestas com os
mesmos caminhos, exceto que trocamos os caminhos
ligando os pares (0,i_l), (j_m,0) e (0,i_p), (j_q,0)
pelos que ligam os pares (0,i_l), (j_q,0) e (0,i_p),
(j_m,0). Mas esse sistema de caminhos está sendo
contado numa outra parcela do determinante, com todos
os fatores iguais, exceto os termos a_{lm} e a_{pq}
que são substituídos por a_{lq} e a_{pm}. Mas o sinal
da permutação está trocado nessa parcela, já que
fizemos uma inversão, então esse sistema de caminhos
aparece cortado. Note que a escolha dessa inversão não
tem ponto fixo e é bijetiva, logo *todos* os caminhos
com interseção se anulam no determinante, e o
resultado segue, já que tal inversão não se aplica a
sistemas de caminhos sem interseção.


 Que tal?

 []'s
 Shine




 __
 Yahoo! Mail - PC Magazine Editors' Choice 2005
 http://mail.yahoo.com









This message was sent using IMP, the Internet
Messaging Program.





__
Do You Yahoo!?
Tired of spam?  Yahoo! Mail has the best spam protection around
http://mail.yahoo.com

Re: [obm-l] Livro Geometria Plana e Espacial

2006-04-15 Por tôpico Igor Castro



por acaso a colecao francesa seriam os 3 livros do 
Carronet???


  - Original Message - 
  From: 
  Romel S. França 
  
  To: obm-l@mat.puc-rio.br 
  Sent: Saturday, April 15, 2006 9:49 
  PM
  Subject: Re: [obm-l] Livro Geometria 
  Plana e Espacial
  
  Edgard de Alencar Filho...geometria plana eh branco com uma figura 
  verde no centro se nao me engano..nao precisa fazer todos os problemas mas faz 
  o sufiente para voce acreditar em voce mesmo
  Morgado 1 e 2
  detona eles e voce vai ser muito boa em geometria plana te 
  garanto...tambem pega umas questoes de concurso para voce acreditar no seu 
  desenvolvimento em geometria.
  Depois que voce tiver uma base mais forte pega o:
  Lidisk etambem tem uma colecao francesa que agora nao me lembro o 
  nome...pode encontrar onde vende livros usados...eu lembro que o nome comeca 
  com C...alguem da lista se lembrado nome?
  On 4/12/06, Bruna 
  Carvalho [EMAIL PROTECTED] 
  wrote: 
  
Alguem poderia me indicar alguns livros, 
autores bons para Geometria Plana e Espacial, sem ser os livros da coleção 
fundamentos de matemática elementar. 
  
  

  No virus found in this incoming message.Checked by AVG Free 
  Edition.Version: 7.1.385 / Virus Database: 268.4.1/313 - Release Date: 
  15/4/2006


Re: [obm-l] limite

2006-02-10 Por tôpico Igor Castro



Basta notar que a_n é estritamente crescente e 
limitada..
[]´s
Igor

  - Original Message - 
  From: 
  Marcelo Salhab 
  Brogliato 
  To: obm-l@mat.puc-rio.br 
  Sent: Friday, February 10, 2006 5:34 
  AM
  Subject: Re: [obm-l] limite
  
  a1 = 300
  b1 = 200 + 0,3 a1
  
  a2 = 300 + 0,3 b1
  b2 = 200 + 0,3 a2
  
  a_n = 300 + 0,3 b_(n-1)
  b_n = 200 + 0,3 a_n
  
  substituindo a_n em b_n, temos:
  b_n = 200 + 0,3 [ 300 + 0,3 b_(n-1) 
]
  b_n = 200 + 90 + 0,09 b_(n-1)
  b_n = 290 + 0,09 b_(n-1)
  
  Supondo que b_n converge, temos que lim b_n = lim 
  b_(n-1).. assim:
  lim b_n = 290 + 0,09 lim b_n
  lim b_n = 290 / [1 - 0,09] = 290 / 0,91 = 318,68 
  = 318 alunos
  
  lim a_n = 300 + 0,3 lim b_n = 300 + 0,3 * 318,68 
  = 395,60 = 395 alunos
  
  faltou provar que as series convergem.. mas nao 
  eh dificil.. olhe:
  se provarmos que b_n converge, então, 
  necessariamente, a_n converge...
  fica como exercicio provar que b_n 
  converge..
  
  abraços,
  Salhab
  
- Original Message - 
From: 
Klaus Ferraz 
To: obm-l@mat.puc-rio.br 
Sent: Thursday, February 09, 2006 8:55 
PM
Subject: [obm-l] limite
Uma faculdade recebe todos os anos 300 alunos novos no 
primeiro semestre e 200 alunos novos no segundo semestre. 30% dos alunos sao 
reprovados no primeiro periodo e repetem o periodo no semestre seguinte. 
Sendo an e bn respectivamente os numeros de alunos do 
primeiro periodo no primeiro e segundo semestres do ano n, calcule 
lim(n--infinito) an e lim (n--infinito)bn. 


Yahoo! SearchDê uma espiadinha e saiba tudo sobre o Big 
Brother Brasil.
  
  

  No virus found in this incoming message.Checked by AVG Free 
  Edition.Version: 7.1.375 / Virus Database: 267.15.4/255 - Release Date: 
  9/2/2006


Re: [obm-l] Intervalos

2006-02-10 Por tôpico Igor Castro



Na primeira parte podemos escrever(jah que bn0 
pra todo n):
a1Beta*b1
a2Beta*b2



anBeta*bn
Faça o mesmo para Alfa(só botar o sinal de 
)

Some tudo e vc tem a primeira 
demonstração.

Na segunda use o mesmo raciocínio e multiplique 
cada linha i por ti.

[]´s
Igor

  - Original Message - 
  From: 
  Klaus 
  Ferraz 
  To: obm-l@mat.puc-rio.br 
  Sent: Friday, February 10, 2006 8:00 
  PM
  Subject: [obm-l] Intervalos
  Se a1/b1,a2/b2,,an/bn pertencem ao intervalo (alpha,beta) e 
  b1,b2,,bn sao positivos prove que (a1+a2+...+an)/(b1+b2++bn) pertence 
  a (alpha,beta). Nas mesmas condicoes se t1,t2,.,tn E R+, prove que 
  (t1a1+t2a2+...+tnan)/(t1b1+t2b2++ tnbn) também pertence ao intervalo 
  (alpha,beta).
  
  
  Yahoo! SearchDê uma espiadinha e saiba tudo sobre o Big 
  Brother Brasil.
  
  

  No virus found in this incoming message.Checked by AVG Free 
  Edition.Version: 7.1.375 / Virus Database: 267.15.4/255 - Release Date: 
  9/2/2006


Re: [obm-l] limite

2006-02-09 Por tôpico Igor Castro



ok... Vamos lá
Podemos encarar a relação da seguinte 
forma...
a_n+1 = 300 + (30/100)*b_n
b_n+1 = 200 + (30/100)*a_n
onde a_1 = 300 e b_1 = 200 + 
(30/100)*300
daí vc tira que a_n = 360 + 
(9/100)a_n-2
que é claramente estritamente crescente e 
tambémlimitada(basta tomar a_n=400 e ver que a_n-2  400, 
absurdo).
Então vc pode admitir que lim(a_n) = L e daí 
aplicar na equação: L = 360 + (9/100)*L
91L = 360*100 == L = 395,6 , ou seja, lim (a_n) 
= 395 alunos "inteiros" :P
Vale o mesmo raciocíniopra b_n.. Espero nao 
ter errado nda..
[]´s
Igor

  - Original Message - 
  From: 
  Klaus 
  Ferraz 
  To: obm-l@mat.puc-rio.br 
  Sent: Thursday, February 09, 2006 8:55 
  PM
  Subject: [obm-l] limite
  Uma faculdade recebe todos os anos 300 alunos novos no primeiro 
  semestre e 200 alunos novos no segundo semestre. 30% dos alunos sao reprovados 
  no primeiro periodo e repetem o periodo no semestre seguinte. Sendo 
  an e bn respectivamente os numeros de alunos do primeiro 
  periodo no primeiro e segundo semestres do ano n, calcule lim(n--infinito) 
  an e lim (n--infinito)bn.
  
  
  Yahoo! SearchDê uma espiadinha e saiba tudo sobre o Big 
  Brother Brasil.


Re: [obm-l] limite

2006-02-09 Por tôpico Igor Castro



Só alguns comentários construtivos eu 
espero...
Acho que não é muito didático observar uma relação 
dos "an´s" iniciais(casos particulares)e , sem demonstração, afirmar que a 
relação é sempre válida para todo n...(de qualquer forma não é nem mesmo uma 
solução).
Acho que o arredondamento para baixo faz mais 
sentido pois estamos interessados no número inteiro de alunos(de fato o numero 
de alunos alcança 395, mas nunca 396).
[]´s
Igor


  - Original Message - 
  From: 
  saulo 
  nilson 
  To: obm-l@mat.puc-rio.br 
  Sent: Thursday, February 09, 2006 10:21 
  PM
  Subject: Re: [obm-l] limite
  
  a1 = 300
  b1 =200+0.3*300
  
  a2 = (200+0.3*300)*0.3 +300= 0.3*200 +300 +0.3^2*300
  b2 = 200 + ((200+0.3*300)*0.3 +300)*0.3 = 200 + 200*0.3^2 
  +300*0.3^3+300*0.3
  
  a3= (200 + ((200+0.3*300)*0.3 +300)*0.3)*0.3 +300= 200*0.3 +200*0.3^3 
  +300*0.3^4 +300*0.3^2 + 300 
  b3= 200+ ((200 + ((200+0.3*300)*0.3 +300)*0.3)*0.3 +300)*0.3= 200 
  +0.3^2*200+200*0.3^4 + 300*0.3^5+300*0.3^3+300*0.3
  
  bn = 200*( 0.3^2n -1)/(0.3^2 -1) + 300*0.3 (0.3^2n -1)/(0.3^2-1)
  an = 200*0.3*(0.3^(2n-2) -1)/(0.3^2-1) + 300*(0.3^2n -1)/(0.3^2-1)
  fazendo o limite para n tendendo ao infinito
  binfinito= 200/(1-0.3^2) +300*0.3/(1-0.3^2)=319 alunos
  ainfinito= 200*0.3/(1-0.3^2) + (300)/(1-0.3^2)=396 alunos
  
  lembrando que a soma tem que dar um numero natural, aproximei para 
  mais
  
  
  
  
  On 2/9/06, Klaus 
  Ferraz [EMAIL PROTECTED] 
  wrote: 
  Uma 
faculdade recebe todos os anos 300 alunos novos no primeiro semestre e 200 
alunos novos no segundo semestre. 30% dos alunos sao reprovados no primeiro 
periodo e repetem o periodo no semestre seguinte. Sendo an e bn 
respectivamente os numeros de alunos do primeiro periodo no primeiro e 
segundo semestres do ano n, calcule lim(n--infinito) an e lim 
(n--infinito)bn. 


Yahoo! SearchDê uma espiadinha e saiba tudo sobre o Big Brother Brasil. 


  
  

  No virus found in this incoming message.Checked by AVG Free 
  Edition.Version: 7.1.375 / Virus Database: 267.15.1/250 - Release Date: 
  3/2/2006


Re: [obm-l] Teoria dos Numeros[off - topic]

2006-01-26 Por tôpico Igor Castro



na www.amazon.com



  - Original Message - 
  From: 
  Klaus 
  Ferraz 
  To: obm-l@mat.puc-rio.br 
  Sent: Thursday, January 26, 2006 7:00 
  PM
  Subject: Re: [obm-l] Teoria dos 
  Numeros[off - topic]
  
  Vlw. Onde consigo esse livro, POWER 
  PLAY de EDWARD J. BARBEAU da 
  MAA Carlos Victor [EMAIL PROTECTED] 
  escreveu: 
  Olá 
Klauss ,(x+1)^3 - x^3 = y^2 , onde 3(2x+1)^2 = 
(2y-1)(2y+1) . Observe que podemos concluir 
que :a) Ou 2y-1 = a^2 e 2y+1 = 
3b^2 b) Ou 2y-1 = 3c^2 e 2y+1 = 
d^2 .Observe que 
3b^2 = a^2 +2 é a única que 
pode ocorrer e, como a é 
ímpar , podemos escrever a = 2t 
+1 e 4y = 2(a^2+1) 
implicando y = t^2 + (t+1)^2 , 
ok ?OBS : (1) Esta questão se 
encontra no Livro POWER PLAY 
de EDWARD J. BARBEAU da MAA ; inclusive 
com a solução acima (2) O 
interessante é que para 3x^2+3x +1 =y^2 
tem para solução geral :x1 = 
4y+7x+3 e y1 = 7y+12x+6 com 
x e y conhecidos . Exemplo : x1 = 104 e 
y1 =181 ; Lindo não é ?[]´s 
Carlos VictorAt 20:23 24/1/2006, Klaus Ferraz 
wrote:
Mostre que a diferença entre os 
  cubos de dois numeros inteiros consecutivos é igual ao quadrado de um 
  inteiro, entao esse inteiro é igual a soma dos quadrados ! de dois 
  inteiros consecutivos.Ex: 8^3-7^3=169. 
  2^2+3^2=13.Grato.Yahoo! doce lar. Faça 
  do Yahoo! sua homepage. 
  
  
  Yahoo! doce lar. Faça 
  do Yahoo! sua homepage.
  
  

  No virus found in this incoming message.Checked by AVG Free 
  Edition.Version: 7.1.375 / Virus Database: 267.14.22/238 - Release Date: 
  23/1/2006


Re:[obm-l] UMA DUVIDA E DOIS PROBLEMAS DA OBM

2006-01-15 Por tôpico Igor Castro



Fala salhab.. curtindo as férias?!
Bem.. pensei nesse problema um pouco.. nao me veio 
nada razoavel e eu desisti :P
Bem, em algum lugar ai vc disse que bastaria 
resolver uma "série" para os termos de x^3, x^2 e etc.. mas o polinomio P^n(x) é 
de gráu maior que 3 né?? o gráu vai depender de n eu acredito e então teriamos 
que controlar cada coeficiente... Bem, meu palpite é de que de alguma forma vc 
consegue colocar fatores em P^n(particularmente nos seus coeficientes)à 
medida que vc aumente o n. Enfim, não consigo sair do lugar.. mas gostaria de 
ver uma solução pra este problema também...
[]´s
Igor

  - Original Message - 
  From: 
  Salhab [ k4ss ] 
  
  To: obm-l 
  Sent: Sunday, January 15, 2006 5:16 
  AM
  Subject: Re:[obm-l] UMA DUVIDA E DOIS 
  PROBLEMAS DA OBM
  
  
  Na 2a. questão, só conclui que preciso provar que |f(p) - f(q) = |p - 
  q|. Mas não consegui faze-lo. Isto é, quase não sai do lugar. :)
  
  Já na 1a. questão, pensei o seguinte, para valer para todo X, então, tem 
  que existir algum N, tal que em p^N(x) - xseja possivel colocar o 101 em 
  evidencia. Entao, o termo independente tem que ser 0 ou um multiplo de 
  101.
  Seja a_n o termo independente de p^n(x), então:
  a_1 = 1
  a_2 = 1 - 2*1 + 14*1 + 1*1 = 14
  
  pois em (p(x))^n o unico termo sem variavel será o 1, entao, (p(x))^3 + 
  14(p(x))^2 - 2p(x) basta analisar para verificar o a_2.
  Já para o a_3, teremos:
  
  a_3 = 1 - 2*a_2 + 14*(a_2)^2 + (a_2)^3
  .
  .
  .
  a_n = 1 - 2*a_(n-1) + 14*(a_(n-1))^2 + (a_(n-1))^3
  
  Agora, é necessário encontrar um valor de n para que a_n seja 0 ou um 
  multiplo de 101.
  Acredito que fazendo uma analise, é possível encontrar series para os 
  termos de x^3, x^2 e x... mas acho que basta o de x, visto que p^N(x) - x é 
  divisivel por 101. Logo, p^N(x) tem que ter o termo x com 
  coeficiente(101*k + 1), onde k é inteiro.
  Penso em algumas possibilidades... resolvendo a sequencia que encontrei, 
  podemos testar o valor de n nas sequencias do x^3, x^2 e x.. e verificar a 
  validade, o que provaria o pedido.
  Também é possivel encontrar o valor para qquer uma das outras sequencias 
  e apenas testa-lo nas outras.
  
  Espero ter ajudado, gostaria de ideias para continuar.. mesmo que por 
  outra linha de raciocinio.
  
  Haaa.. mesmo que esquecam esse meu raciocinio, gostaria muito que alguem 
  me ensina-se como resolver esse tipo de sequencia.. é um recorrencia de 1a. 
  ordem, mas não linear.. nunca vi nada parecido.
  
  Um abraço,
  Salhab
  
  
  
   - Duvida: na solução do problema 6 da OBM - Nivel U - Segunda Fase, 
  que aparece na Eureka 22 está escrito: "Temos ainda |a'(t)| é menor que ou 
  igual a 2 para todo "t", donde o comprimento da curva "a" é menor ou igual a 
  4pi". Alguém poderia me explicar por que isso é válido. 
   
   - Já faz algum tempo que postei os seguintes problemas da obm. Como 
  ainda não apareceu nenhuma solução estou postando-os novamente. 
   
   1- (OBM 1996) Seja p(x) o polinomio x^3 + 14x^2 - 2x + 1. Defina 
  p^n(x) como 
   p(p^(n -1)(x)). Mostre que existe um inteiro N tal que p^N(x) - x é 
  divisivel por 101 para todos os inteiros x. 
   
   2- (OBM 2001 - Nivel U) Seja D o conjunto de pontos de R^2 com |p| 
  menor que ou igual a 1. Seja f : D = D uma função sobrejetora tal que 
  
   |f(p) - f(q)| é menor que ou igual a |p - q| para quaisquer p, q de 
  D. Prove que 
   |f(p) - f(q)| = |p - q|. 
   ( |(x,y)| = sqrt(x^2 + y^2) ) 
   
   - obs: Uma solução para o problema 2 encontra-se na Eureka 13. No 
  entanto, é definida uma função f~ "composição de rotação com espelhamento que 
  coincide com f nos pontos p, q, -p e -q". O que me garante a existência de tal 
  função? Por quê ela é uma bijeção? Existe uma solução alternativa que não 
  utilize tal conceito e nem teoria das medidas? 
   
   
   
   - 
   Yahoo! doce lar. Faça do Yahoo! sua homepage.
  
  

  No virus found in this incoming message.Checked by AVG Free 
  Edition.Version: 7.1.371 / Virus Database: 267.14.17/229 - Release Date: 
  13/1/2006


Re: [obm-l] Divisibilidade

2006-01-14 Por tôpico Igor Castro



A técnica que se usa é escrever exatamente o que 
está escrito...
"Um número xdividido por 7 dá resto 
2" ou "O número xé um multiplo de 7, mais 2"
matemáticamente:

x = 7m + 2 ;m 
natural

coma 
outra informação:
x = 2n +1 ; n 
natural

mas vc quer que 
apareça x= 14k + r. Então vc multiplica a primeira eq. por 2e a segunda 
eq. por 7:
2x=14m + 
4
7x=14n + 
7

subtraia a segunda 
eq. de 3 vezes a primeira eq: 7x - 6x = 14n - 3.14m +7 - 12
ou seja: x = 14(n - 
3m) -5
aqui vc deve deixar 
o resto positivo, pra isso vc faz n-3m=k+1 e obtem: x = 14k + 9 . Logo o resto 
da divisao por 14 é 9.

ps: como isso vale 
sempre.. vc tbm pode chutar uns numeros que satisfaçam o que ele falou e ver o 
seu resto por 14. Daí vc vÊ que o 9 satisfaz o que o prob disse e tbm sabe que o 
resto de 9 por 14 é o próprio 9.



  - Original Message - 
  From: 
  Bruna Carvalho 
  To: obm-l@mat.puc-rio.br 
  Sent: Saturday, January 14, 2006 3:49 
  PM
  Subject: [obm-l] Divisibilidade
  Um número dividido por 7 dá 
  resto 2 e dividido por 2 da resto 1. Determinar o resto da divisão desse 
  número por 14.Alguem pode me 
  ajudar com essa questão, e me explicar a tecnica que se ultiliza para resolver 
  esse tipo de questão!! 
  
  

  No virus found in this incoming message.Checked by AVG Free 
  Edition.Version: 7.1.371 / Virus Database: 267.14.17/229 - Release Date: 
  13/1/2006


Re: Re:[obm-l] Divisibilidade

2006-01-14 Por tôpico Igor Castro



Falai luiz!! achovc se enganou na linha que 
eu destaquei abaixo.. confira!
Abraços.. 
Igor

  - Original Message - 
  From: 
  Luiz H. 
  Barbosa 
  To: obm-l 
  Sent: Saturday, January 14, 2006 7:23 
  PM
  Subject: Re:[obm-l] Divisibilidade
  
  
  Um número dividido por 7 dá resto 2 e dividido por 2 da resto 1. 
  Determinar 
  o resto da divisão desse número por 14. 
  
  ==
  Bom , se não me engano essa questão foi do ano que eu fiz UFRJ.
  Da pra resolver de muitas formas.
  Vou usar congruencia.
  
  entenda (=) como o sinal de congruencia que são 3 traçinhos !!!
  
  x(=)2(mod7) e x(=)1(mod2) são dados do problema .
  
  Mas podemos escrever 
  2x(=)4(mod14) i,multipliquei tudo por 2 .
  7x(=)7(mod14)ii,multipiquei tudo por 7.
  
  Diminuindo ii - i 
  : 
  
  5x(=)3(mod14)iii
  Somando i + 
  ii
  9x(=)11(mod14) iv
  
  iii - i , fica:
  3x(=)-1(mod14)
  iv - ii , fica:
  2x(=)3(mod14) 
  ESSA**
  
  Diminuindo uma da outra , temos :
  x(=)-4(mod14)
  
  O que significa que o resto de x por 14 é -4.
  MSN : [EMAIL PROTECTED]
  Abraço,
  Luiz H. Barbosa 
  
  
  
  
  
  

  No virus found in this incoming message.Checked by AVG Free 
  Edition.Version: 7.1.371 / Virus Database: 267.14.17/229 - Release Date: 
  13/1/2006


[obm-l] Re: [obm-l] Re: [obm-l] RES: [obm-l] lóg ica: condional e bicondicional

2006-01-07 Por tôpico Igor Castro



Ai que está, a única informação que eu te dei é que 
SE VC PASSAR ENTÃO TE DOU UM CARRO. Não mencionei o que eu 
faria caso vc nao passasse..Assim posso optar por te dar o carro ou não, que 
continuarei falando a verdade.

Já no segundo problema(o com =) aí 
sim esse conectivo indica o caso "se e somente se", ou seja, a proposicao 
ficaria "Te dou um carro, se e somente se, você passar de ano." 
entendeu?

  - Original Message - 
  From: 
  Bruna Carvalho 
  To: obm-l@mat.puc-rio.br 
  Sent: Saturday, January 07, 2006 1:10 
  PM
  Subject: Re: [obm-l] Re: [obm-l] RES: 
  [obm-l] lógica: condional e bicondicional
  Igor na item c, eu só ganho o carro se eu passar de ano. esse 
  não é a condição para ganhar ??
  
  

  No virus found in this incoming message.Checked by AVG Free 
  Edition.Version: 7.1.371 / Virus Database: 267.14.15/223 - Release Date: 
  6/1/2006


[obm-l] Re: [obm-l] RES: [obm-l] lógica: condional e bicondicional

2006-01-05 Por tôpico Igor Castro



a) nesse caso considera a proposição 1 como sendo 
"vc passar de ano" e a 2 como "te dou um carro". Utilizando o - temos "se vc 
passar de ano então te dou um carro)
Agora vc analisa os casos:
a) V - V ... significa q vc passou de ano e eu 
te dei um carro(ok,falei a verdade - V)
b) V - F ... significa que vc passou de ano e 
eu nao te dei um carro (eu menti - F )
c) F - V  significa que vc nao passou de 
ano e eu te dei um carro (ok, pois já q vc nao passou posso fazer o q eu quiser 
que vou estar falando a verdade- V)
d) igual ao (c)

agora tenta usar essa mesma ideia no exercicio 
seguinte usando o "se, somente se" ok?


  - Original Message - 
  From: 
  Bruna Carvalho 
  To: obm-l@mat.puc-rio.br 
  Sent: Thursday, January 05, 2006 1:43 
  PM
  Subject: Re: [obm-l] RES: [obm-l] lógica: 
  condional e bicondicional
  a) No caso do "-":1) V - V = V2) V 
  - F = F3) F - V = V4) F - F = Vpq o item 2 é 
  Fpq o item 3 é Vse possível gostaria de alguém explicasse a tabela 
  inteira ou me indicasse onde eu poderia consultar. b) No caso do 
  "-":1)V - V = V2)V - F = F3)F 
  - V = F4)F - F = Vessa eu não entendi nda!!
  
  

  No virus found in this incoming message.Checked by AVG Free 
  Edition.Version: 7.1.371 / Virus Database: 267.14.12/220 - Release Date: 
  3/1/2006


Re: [obm-l] Mais um Legal

2005-11-26 Por tôpico Igor Castro

Fala ai!
só consegui uma demo(meio inutil, reconheço) quando o triangulo é 
acutangulo..
fora isso nao consigo pensar em nda a nao ser fazer umas contas imensas e 
usar alg desigualdade auxiliar..

vc tem uma solução legal?


- Original Message - 
From: Marcos Martinelli [EMAIL PROTECTED]

To: obm-l@mat.puc-rio.br
Sent: Saturday, November 26, 2005 9:14 PM
Subject: [obm-l] Mais um Legal



  Sejam A,B e C ângulos de um triângulo ABC.
  Provar que 1/sen(A)+1/sen(b)=8/[3+2*cos(C)].

=
Instruções para entrar na lista, sair da lista e usar a lista em
http://www.mat.puc-rio.br/~nicolau/olimp/obm-l.html
=


--
No virus found in this incoming message.
Checked by AVG Free Edition.
Version: 7.1.362 / Virus Database: 267.13.4/176 - Release Date: 20/11/2005









___ 
Faça do Yahoo! sua página inicial. 
http://br.yahoo.com/homepageset.html 


=
Instruções para entrar na lista, sair da lista e usar a lista em
http://www.mat.puc-rio.br/~nicolau/olimp/obm-l.html
=


[obm-l] Outras Soluções - IME 2006

2005-10-26 Por tôpico Igor Castro
Uma pessoal do IME também ta resolvendo a prova...(inclusive já quase 
terminaram a de física).

http://www.gabaritando.cjb.net/
[]?s
Igor 







___ 
Promoção Yahoo! Acesso Grátis: a cada hora navegada você

acumula cupons e concorre a mais de 500 prêmios! Participe!
http://yahoo.fbiz.com.br/

=
Instruções para entrar na lista, sair da lista e usar a lista em
http://www.mat.puc-rio.br/~nicolau/olimp/obm-l.html
=


Re: [obm-l] prop-arquimediana

2005-09-26 Por tôpico Igor Castro



n=b+1 pois (b+1).a = ab + a  b
ou maior...
certo?
Abraços
Igor...

  - Original Message - 
  From: 
  reibellini 
  To: obm-l@mat.puc-rio.br 
  Sent: Monday, September 26, 2005 8:01 
  PM
  Subject: [obm-l] prop-arquimediana
  
  


  


Mostre que N 
possui propriedade arquimediana, 
ou seja, 
dados a,b pertencentes a N, 0ab, 

existe n 
pertencente a N tal que nab.

Grato por qualquer 
ajuda.

Cordialmente,
Jerry

da uma olhada no elon prop arquimediana , no livro maior , o que 
você quer dizer com N, (naturais) ?fiquei em duvida , mande um e-mail de 
novo valeu 
reinaldo - ufjf

-- 
  
  

  No virus found in this incoming message.Checked by AVG 
  Anti-Virus.Version: 7.0.344 / Virus Database: 267.11.6/111 - Release Date: 
  23/9/2005


[obm-l] Re: [obm-l] Livro de Cálculo

2005-09-24 Por tôpico Igor Castro



Eu não conheço o livro. Mas já ouvi falar MUITO bem 
dele... O que você achou ?
[]´s
Igor

  - Original Message - 
  From: 
  Guilherme Neves 
  To: obm-l@mat.puc-rio.br 
  Sent: Saturday, September 24, 2005 10:08 
  AM
  Subject: [obm-l] Livro de Cálculo
  
  
  Alguém conhece o livro Differential and Integral Calculus , 
  V.1- Richard Courant? Meu primo o comprou pra mim nos EUA e gostaria de saber 
  se ele eh bom.. 
  sugestões..= 
  Instruções para entrar na lista, sair da lista e usar a lista em http://www.mat.puc-rio.br/~nicolau/olimp/obm-l.html 
  =
  
  

  No virus found in this incoming message.Checked by AVG 
  Anti-Virus.Version: 7.0.344 / Virus Database: 267.11.6/111 - Release Date: 
  23/9/2005


[obm-l] Re: [obm-l] demonstração

2005-08-28 Por tôpico Igor Castro



Sinceramente eu não entendi o que vc fez ali com as 
constantes, poderia explicar melhor

Acho que dá pra pensar assim tbm(alguem me corrija 
se estiver errado...)
Se a(xo) + b(yo) +c = 0
apartir de (xo,yo) vc obtém (xo + bt, yo - at) tal 
que
a(xo + bt) + b( yo - at) + c = 0
logo, a cada t inteiro vc tem novos ptos de 
coordenadas inteiras. Como Z é infinito
[]´s
Igor

  - Original Message - 
  From: 
  Renato 
  G Bettiol 
  To: obm 
  Sent: Sunday, August 28, 2005 6:07 
  PM
  Subject: [obm-l] demonstração
  
  
  Caríssimos,
  Tal problema estava na terceira fase da olimpíada de matemática da Unicamp, 
  elaborada por A. C. Patrocínio.
  "Sejam a, b e c números naturais não-nulos e suponha que a reta ax+by+c=0 
  passe pelo ponto (xo,yo) com xo e yo inteiros. Mostre que a mesma reta passa 
  por infinitos pontosde coordenadasinteiras."
  Não sei se está correta minha resolução, que foi a de isolar xo e yo, 
  afinal são raízes da equação. Somei então duas constantes inteiras, k' e k'', 
  uma a xo e outra a yo. Isolando novamente xo e yo, acrescidos agora desta 
  constante, e colocando-os de volta na equação, obtem-se
  a(xo+2k') + b(yo+2k'') + c = 0
  o que nitidamente mostra que (xo+2k';yo+2k'') é outra raiz da equaçao, 
  outro ponto de inteiros pelo quala reta passa. E assim, por indução, 
  temos que ha infinitos pontos de coordenadas inteiras que satisfazem 
  ax+by+c=0.
  Será que poderiam comentar a resolução? Haveria uma interpretação 
  geométrica? Pensei em semelhança de triângulos, para fazer a mesma 
  demonstraçao de um modo pouco mais elegante...
  Abraço a todos, agradeço previamente,
  
  Renato
  
  

  No virus found in this incoming message.Checked by AVG 
  Anti-Virus.Version: 7.0.344 / Virus Database: 267.10.16/83 - Release Date: 
  26/8/2005


Re: [obm-l] integral

2005-06-23 Por tôpico Igor Castro

Se n me engano, int[ (1-4y)^(1/2)dy ] = -(1/6)(1-4y)^(3/2)
[]´s
Igor Castro

- Original Message - 
From: Rodrigo Soares [EMAIL PROTECTED]

To: obm-l@mat.puc-rio.br
Sent: Thursday, June 23, 2005 9:11 PM
Subject: [obm-l] integral



Alguem me de a resoluçaum desse problema por favor:

Integral da raiz quadrada de 1 - 4y

preciso dele pra hj
desculpe por naum botar os simbolos.. eh q eu naum sei como coloca-los...
[]s

=
Instruções para entrar na lista, sair da lista e usar a lista em
http://www.mat.puc-rio.br/~nicolau/olimp/obm-l.html
=


--
No virus found in this incoming message.
Checked by AVG Anti-Virus.
Version: 7.0.323 / Virus Database: 267.7.8/22 - Release Date: 17/6/2005








___ 
Yahoo! Acesso Grátis - Internet rápida e grátis. 
Instale o discador agora! http://br.acesso.yahoo.com/


=
Instruções para entrar na lista, sair da lista e usar a lista em
http://www.mat.puc-rio.br/~nicolau/olimp/obm-l.html
=


[obm-l] RE: [obm-l] livros de cálculo

2005-04-21 Por tôpico Igor Castro
Se vc quiser um livro bem completo e formal(talvez dificil? depende do pto 
de vista), usa o Calculus - Tom M. Apostol(Vol 1 e 2). É um livro MUITO bom 
: )
Não acho que seja dificil encontrá-lo a venda...
[]?s
Igor

- Original Message - 
From: [EMAIL PROTECTED]
To: obm-l@mat.puc-rio.br
Sent: Thursday, April 21, 2005 4:15 PM
Subject: [obm-l] RE: [obm-l] livros de cálculo


eu gosto do Stewart
's'-- Mensagem Original --
''Date: Thu, 21 Apr 2005 16:06:56 -0300
''From: Thiago Addvico [EMAIL PROTECTED]
''To: obm-l@mat.puc-rio.br
''Subject: [obm-l] livros de cálculo
''Reply-To: obm-l@mat.puc-rio.br
''
''
''olá
''
''estou cursando calculo 1 na universidade e gostaria de pedir uma
''recomendação de livro, pois lá estudamos com o anton e ele usa muito
a
''calculadora, ou softwares gráficos.
''
''Um abraço!
''
''=
''Instruções para entrar na lista, sair da lista e usar a lista em
''http://www.mat.puc-rio.br/~nicolau/olimp/obm-l.html
''=

=
Instruções para entrar na lista, sair da lista e usar a lista em
http://www.mat.puc-rio.br/~nicolau/olimp/obm-l.html
=
--
No virus found in this incoming message.
Checked by AVG Anti-Virus.
Version: 7.0.308 / Virus Database: 266.10.1 - Release Date: 20/4/2005


=
Instruções para entrar na lista, sair da lista e usar a lista em
http://www.mat.puc-rio.br/~nicolau/olimp/obm-l.html
=


[obm-l] Re: [obm-l] Como faz ? é legal

2005-03-05 Por tôpico Igor Castro



Seja x=y^2 -  sqrt(x)= |y|.. 
substituindo
|y| + m = y^2 e analisando os casos de y0 e 
y0 você chega que y1 = (1 +sqrt(1+4m))/2e y2= (-1 -sqrt(1+4m))/2 

onde |y1|=|y2| e como x=y^2 = |y|^2, x só assume um 
valor( = [(1 +sqrt(1+4m))/2]^2 ) para cada m.
[]´s
Igor

  - Original Message - 
  From: 
  Robÿe9rio Alves 
  To: obm-l@mat.puc-rio.br 
  Sent: Saturday, March 05, 2005 6:03 
  PM
  Subject: [obm-l] Como faz ? é legal
  
  Mostre que, para todo m0, a equação rsqt(x) + m = x tem apenas 
  uma raiz. Como faz ?
  
  
  Yahoo! 
  Acesso Grátis - Internet rápida e grátis. Instale o discador do Yahoo! 
  agora.
  
  

  No virus found in this incoming message.Checked by AVG 
  Anti-Virus.Version: 7.0.308 / Virus Database: 266.6.2 - Release Date: 
  4/3/2005
No virus found in this outgoing message.
Checked by AVG Anti-Virus.
Version: 7.0.308 / Virus Database: 266.6.2 - Release Date: 4/3/2005


[obm-l] Sequência...

2005-03-04 Por tôpico Igor Castro



Como faço???
Seja uma sequência de Xn tal que Lim X(n+1)/X(n) 
(n- inf) = r.
Provar que Lim (Xn)^(1/n) = r no infinito 
também..
Estou com um pouco de dúvida para mostrar que uma 
série converge/diverge.. Alguém pode me dizer os critérios e os métodos pra 
demonstrar essas afirmações??
[]´s
Igor Castro
No virus found in this outgoing message.
Checked by AVG Anti-Virus.
Version: 7.0.300 / Virus Database: 266.6.2 - Release Date: 4/3/2005


Re: [obm-l] oi!

2005-01-10 Por tôpico Igor Castro
Oi. Eu não te conheço, mas vai pro ita, acho que profissionalmente é 
melhor(e não faça isso tarde  d+, tem muita gente na fila de espera do ime 
querendo entrar). Ah,  vai pro ime somente se você achar um pouco + legal 
estudar numa faculdade boa, perto do melhor?(maior?) centro de matemática do 
país e que fica na cidade em que boa parte do MUNDO gostaria de passar as 
férias. Sabe como é, essas coisas de praia, cidade maravilhosa e etc...
A decisão é sua, boa sorte.
Igor Castro

- Original Message - 
From: Kellem :-) 100% SeJ [EMAIL PROTECTED]
To: obm-l@mat.puc-rio.br
Sent: Sunday, January 09, 2005 5:50 PM
Subject: [obm-l] oi!


Oi Gente
Bem, meu nome é Kellem e sou do RJ. Fazia UFF e, agora q passeo pro IME e
pro ITA, vou pra um dos dois, só q, até agora, fico sem dormir sem saber o 
q
fazer, pq tb faço Iniciação Científica no IMPA! Eu não sei se fico pra
arriscar o IMPA... O problema é q gosto muito de matemática, mas acho q 
não
sei mto não (sempre acho q não sei nada e q vou tirar zero). Enfim, tô
aqui, e espero poder ajudar e obter ajuda nos probleminhas, tá?
Alguém daqui me conhece (ou eu conheço??)?
BJão
Kellem

=
Instruções para entrar na lista, sair da lista e usar a lista em
http://www.mat.puc-rio.br/~nicolau/olimp/obm-l.html
=
--
No virus found in this incoming message.
Checked by AVG Anti-Virus.
Version: 7.0.300 / Virus Database: 265.6.8 - Release Date: 3/1/2005


--
No virus found in this outgoing message.
Checked by AVG Anti-Virus.
Version: 7.0.300 / Virus Database: 265.6.8 - Release Date: 3/1/2005
=
Instruções para entrar na lista, sair da lista e usar a lista em
http://www.mat.puc-rio.br/~nicolau/olimp/obm-l.html
=


Re: [obm-l] numero primo?

2004-12-04 Por tôpico Igor Castro



Esse programa só testa os números nesse 
intervalo??? Sabe onde tem o cód fonte?
Ouvi falar a um tempo que descobriram um algoritmo 
pra testar primalidade em um tempo razoável... tinha até o codígo fonte em site. 
Alguém sabe onde posso achar isso?
[]´s
Igor

  - Original Message - 
  From: 
  Johann Peter Gustav Lejeune 
  Dirichlet 
  To: [EMAIL PROTECTED] 
  Sent: Friday, December 03, 2004 4:15 
  PM
  Subject: Re: [obm-l] numero primo?
  
  Voce tem o codigo-fonte?Araray Velho [EMAIL PROTECTED] wrote: 
  Renato,Esse 
número não é primo, pois é divisível por 1, 19, 97, 277, 1843,5263, 
26869 e 510511Podes baixar um programinha que testa se um número é 
primo ou não e,caso não seja, dá a fatoração desse número. O programa 
foidesenvolvido por mim em 1999 e tem apenas 17kB . O endereço 
paradownload é 
http://www.somatematica.com.br/zips/primos.zipAbraços,Araray 
VelhoOn Thu, 2 Dec 2004 22:22:42 -0200, Marcio 
Cohen<[EMAIL PROTECTED]>wrote: Esse número é 
composto... Note que 30*7*11*13*17 = 11*7*11*13*(-2) = 1*11*13*(-2) 
= 10*(-2) = -20 = - 1 (mod 19), e portanto o seu número é divisível 
por 19.. - Original Message 
- From: "Renato Lira" <[EMAIL PROTECTED]> To: 
<[EMAIL PROTECTED]> Sent: Thursday, December 02, 2004! 8:31 
PM Subject: [obm-l] numero primo?   gostaria de 
saber se esse numero é primo, se nao, gostaria de saber  alguma 
fatoracao pra achar ele   2x3x5x7x11x13x17 + 
1 Grato, Renato 
Lira.   
= 
 Instruções para entrar na lista, sair da lista e usar a lista 
em  http://www.mat.puc-rio.br/~nicolau/olimp/obm-l.html 
 
= 
   
= 
Instruções para entrar na lista, sair da lista e usar a lista em 
http://www.mat.puc-rio.br/~nicolau/olimp/obm-l.html 
= 
-- ! Araray Velho[EMAIL PROTECTED]ICQ 20464041MSN 
[EMAIL PROTECTED]=Instruções 
para entrar na lista, sair da lista e usar a lista 
emhttp://www.mat.puc-rio.br/~nicolau/olimp/obm-l.html=
  __Converse com seus 
  amigos em tempo real com o Yahoo! Messenger 
  http://br.download.yahoo.com/messenger/ 
  
  

  No virus found in this incoming message.Checked by AVG 
  Anti-Virus.Version: 7.0.289 / Virus Database: 265.4.4 - Release Date: 
  30/11/2004
No virus found in this outgoing message.
Checked by AVG Anti-Virus.
Version: 7.0.289 / Virus Database: 265.4.4 - Release Date: 30/11/2004


Re: [obm-l] Duvidas

2004-11-19 Por tôpico Igor Castro



para que (x2-5x+7)x+1=1 
temos:

basta 
que x^2 - 5x + 7 = 1 ( 1 elevado a qualquer expoente é 1) - 2 
soluções
ou
x+1 = 
0 e x^2 - 5x + 7 =/= 0 - 1 solução
Logo, 
como as 3 soluções são diferentes, serão 3 soluções. (d)
[]´s
Igor

  - Original Message - 
  From: 
  aryqueirozq 
  To: obm-l 
  Sent: Friday, November 19, 2004 2:59 
  PM
  Subject: [obm-l] Duvidas
  
  
  
  Quantos 
  números reais satisfazem a equação
  (x2-5x+7)x+1=1 
  ?
  
  a) 
  0 
  b) 1 
  c) 2 
  d) 3 
  e) 4 
  
  
  Me 
  ajudem , nesta equação , só estou achando - 1 como resposta( logo uma 
  soluçaõ), mas o gabarito está dando como resposta letra D. Quais são as outras 
  soluções?
  
  
  Agradeço.
  
  

  No virus found in this incoming message.Checked by AVG 
  Anti-Virus.Version: 7.0.289 / Virus Database: 265.3.1 - Release Date: 
  15/11/2004
No virus found in this outgoing message.
Checked by AVG Anti-Virus.
Version: 7.0.289 / Virus Database: 265.3.1 - Release Date: 15/11/2004


Re: [obm-l] IME

2004-10-27 Por tôpico Igor Castro



Me pareceu BEM claro que o desenho era plano. Mesmo assim 
continua sendo uma helicoidal, ele faz um MCU no plano perpendicular a B e 
também desce com V.Cos(Teta).
[]´s
Igor

  - Original Message - 
  From: 
  Bernardo 
  To: [EMAIL PROTECTED] 
  Sent: Wednesday, October 27, 2004 7:14 
  PM
  Subject: Re: [obm-l] IME
  
  
  Hoje a prova tava legal.
  
  Só vi um erro na prova. Na questão 02, ele dava os vetores 
  de indução e o vetor de velocidade formando um angulo "teta"
  
  Mas não falava nada na questão nem deixava indicado na 
  figura se o vetor velocidade estava no plano do papel ou se tinha alguma 
  projeção vertical. Com isso alguns acharam uma helicoidal e outros uma 
  circunferencia.
  
  Eu não sei o que acontece nesses casos, eu na hora nem 
  percebi isso e fiz como se o vetor estivesse no mesmo plano do papel. Qdo saí 
  é que vi que algumas pessoas acharam uma helicoidal. 
  
  O que vcs acham?
  
  Abraços,
  Bernardo
  
  
  ---Outgoing mail is certified Virus Free.Checked 
  by AVG anti-virus system (http://www.grisoft.com).Version: 6.0.782 
  / Virus Database: 528 - Release Date: 
22/10/2004


Re: [obm-l] IME

2004-10-26 Por tôpico Igor Castro
concordo com tudo que vc falou..
a questão 3 acho que deveria ser anulada..
já essa 4 é cruel mesmo, mas é cruel pra todo mundo...
[]´s
Igor

- Original Message - 
From: Ariel de Silvio [EMAIL PROTECTED]
To: [EMAIL PROTECTED]
Sent: Tuesday, October 26, 2004 9:16 PM
Subject: [obm-l] IME


 Olá a todos,

 Começaram hoje as provas do IME. Hoje foi realizada a prova de matemática.
 Lembro que ano passado propuseram na lista resoluções das questões
 diferentes da resoluções dadas pelos cursinhos. Esse ano vão fazer também?

 O Poliedro (www.sistemapoliedro.com.br) está resolvendo. O GPI diz que irá
 resolver também (www.gpi.g12.br). O Poliedro está colocando o enunciado em
 apenas algumas das questões.

 Mas já começo com um pedido, a questão 3. Vou passar direto aqui.

 Sejam a, b, c, d números reais positivos e diferentes de 1. Sabendo que
 log[a](d), log[b](d) e log[c](d) são termos consecutivos de um progressão
 aritmética, demonstre que:
 c^2 = (ac)^log[a](d)

 log[a](d) é log de d na base a

 Só que ninguém que conversei conseguiu chegar nisso. Apenas em:

 c^2 = (ac)^log[a](b)

 Cheguei nisso, e não vejo motivo para b = d

 De resto tiveram questões MUITO simples, outras malvadas e outras
realmente
 difíceis.
 A questão 4 por exemplo dava duas equações de quarto grau, pedia as raizes
 comuns. Porém não tinha raízes comuns! Cruel pra quem tá ali fazendo a
prova
 .

 []s
 Ariel

 =
 Instruções para entrar na lista, sair da lista e usar a lista em
 http://www.mat.puc-rio.br/~nicolau/olimp/obm-l.html
 =


---
Outgoing mail is certified Virus Free.
Checked by AVG anti-virus system (http://www.grisoft.com).
Version: 6.0.782 / Virus Database: 528 - Release Date: 22/10/2004

=
Instruções para entrar na lista, sair da lista e usar a lista em
http://www.mat.puc-rio.br/~nicolau/olimp/obm-l.html
=


Re: [obm-l] OMERJ-2004

2004-10-24 Por tôpico Igor Castro
Muito legal a prova! Parabens!
gostei particularmente das questões 2 e 5(nivel4)
[]´s
Igor

- Original Message - 
From: Fabio Dias Moreira [EMAIL PROTECTED]
To: [EMAIL PROTECTED]
Sent: Sunday, October 24, 2004 12:54 AM
Subject: Re: [obm-l] OMERJ-2004



 Igor Castro said:
  alguém pode postar a prova da olimpiada de matemática do estado do
  RJ(foi hj)??? último nível - 3 ano +++
  []´s
  Igor
  [...]

 As provas da OMERJ, níveis 3 e 4 já estão no ar com as suas respectivas
 soluções da banca, na seção de provas do site da OMERJ
 (http://www.omerj.com.br/).

 (A solução da questão do polinômio, infelizmente, foi digitada
 incorretamente; as únicas soluções são (x+1)(x-4) e o seu simétrico. Uma
 versão [EMAIL PROTECTED] do gabarito deve ir ao ar dentro de algumas horas.)

 []s,

 -- 
 Fábio ctg \pi Dias Moreira
 Membro da Comissão de Olimpíadas de Matemática do Estado do Rio de Janeiro



---
Outgoing mail is certified Virus Free.
Checked by AVG anti-virus system (http://www.grisoft.com).
Version: 6.0.782 / Virus Database: 528 - Release Date: 22/10/2004

=
Instruções para entrar na lista, sair da lista e usar a lista em
http://www.mat.puc-rio.br/~nicolau/olimp/obm-l.html
=


[obm-l] OMERJ-2004

2004-10-23 Por tôpico Igor Castro



alguém pode postar a prova da olimpiada de 
matemáticado estado do RJ(foi hj)???
último nível - 3 ano +++
[]´s
Igor

---Outgoing mail is certified Virus 
Free.Checked by AVG anti-virus system (http://www.grisoft.com).Version: 6.0.782 / 
Virus Database: 528 - Release Date: 22/10/2004


Re: [obm-l] OBM 2004 - NIVEL 3

2004-10-18 Por tôpico Igor Castro
Tem uma linha do seu quadrado com 5 alg diferentes... Como as outras tem 4..
acho que isso não mostra nada..
Enfim, eu achei um que só tinha linhas e colunas com exatos 5 alg
diferentes.. Logo, n 6, mas não consegui provar que os valores abaixo de 5
não são(se é que essa é a resposta).. só cheguei que n=5 e disse que o
valor máximo é 5..
É... a prova esse ano tava MUITO mais dificil que o ano passado.. não tem
nenhuma questão que se possa dizer que foi pro cara não zerar.. Acho que não
fiz nenhuma inteira também..
[]´s
Igor
ps: Alguem pode dizer a solução da 5??

- Original Message - 
From: Paulo Rodrigues [EMAIL PROTECTED]
To: [EMAIL PROTECTED]
Sent: Monday, October 18, 2004 10:54 AM
Subject: Re: [obm-l] OBM 2004 - NIVEL 3 (2)


 Tinha esquecido uma linha:

 445566
 001144
 001166
 001177
 445566
 445566
 2233885588
 223388
 223399
 445566

 - Original Message -
 From: Paulo Rodrigues [EMAIL PROTECTED]
 To: [EMAIL PROTECTED]
 Sent: Monday, October 18, 2004 9:25 AM
 Subject: Re: [obm-l] OBM 2004 - NIVEL 3


 Acho que o exemplo abaixo está certo. Confira por favor

 445566
 001144
 001166
 001177
 445566
 445566
 2233885588
 223388
 445566
 - Original Message -
 From: Igor Oliveira [EMAIL PROTECTED]
 To: [EMAIL PROTECTED]
 Sent: Monday, October 18, 2004 8:55 AM
 Subject: [obm-l] OBM 2004 - NIVEL 3


   E aí pessoal, como vocês foram na 3ª FASE (nível 3) desse ano? Eu achei
a
 prova
 desse ano muito mais dificil do que a do ano passado Acho que não
 consegui
 fazer nenhuma questão inteira...

   Uma pergunta...: qual é a resposta da 4ª Questão, aquela do tabuleiro?
Eh
 n=4 ou
 n=5? Não consegui achar um tabuleiro onde n=4. Se a resposta for n=4
mesmo,
 me
 digam aí como preencher o tabuleiro.


   Igor


 =
 Instruções para entrar na lista, sair da lista e usar a lista em
 http://www.mat.puc-rio.br/~nicolau/olimp/obm-l.html
 =


 ---
 Outgoing mail is certified Virus Free.
 Checked by AVG anti-virus system (http://www.grisoft.com).
 Version: 6.0.778 / Virus Database: 525 - Release Date: 15/10/2004

 =
 Instruções para entrar na lista, sair da lista e usar a lista em
 http://www.mat.puc-rio.br/~nicolau/olimp/obm-l.html
 =

 =
 Instruções para entrar na lista, sair da lista e usar a lista em
 http://www.mat.puc-rio.br/~nicolau/olimp/obm-l.html
 =


---
Outgoing mail is certified Virus Free.
Checked by AVG anti-virus system (http://www.grisoft.com).
Version: 6.0.778 / Virus Database: 525 - Release Date: 15/10/2004

=
Instruções para entrar na lista, sair da lista e usar a lista em
http://www.mat.puc-rio.br/~nicolau/olimp/obm-l.html
=


Re: [obm-l] OBM 2004 - NIVEL 3

2004-10-18 Por tôpico Igor Castro
É! me confundi... contei 5 alg diferentes umas 10 vezes na sétima linha :P
mas enfim.. ok.. 4 alg diferentes sempre... essa era a resposta então?
como provar que não tem um quadrado com 3? 2?
[]´s
Igor Castro

- Original Message - 
From: Paulo Rodrigues [EMAIL PROTECTED]
To: [EMAIL PROTECTED]
Sent: Monday, October 18, 2004 3:38 PM
Subject: Re: [obm-l] OBM 2004 - NIVEL 3


 Qual linha?
 - Original Message -
 From: Igor Castro [EMAIL PROTECTED]
 To: [EMAIL PROTECTED]
 Sent: Monday, October 18, 2004 12:18 PM
 Subject: Re: [obm-l] OBM 2004 - NIVEL 3


 Tem uma linha do seu quadrado com 5 alg diferentes... Como as outras tem
4..
 acho que isso não mostra nada..
 Enfim, eu achei um que só tinha linhas e colunas com exatos 5 alg
 diferentes.. Logo, n 6, mas não consegui provar que os valores abaixo de
5
 não são(se é que essa é a resposta).. só cheguei que n=5 e disse que o
 valor máximo é 5..
 É... a prova esse ano tava MUITO mais dificil que o ano passado.. não tem
 nenhuma questão que se possa dizer que foi pro cara não zerar.. Acho que
não
 fiz nenhuma inteira também..
 []´s
 Igor
 ps: Alguem pode dizer a solução da 5??

 - Original Message -
 From: Paulo Rodrigues [EMAIL PROTECTED]
 To: [EMAIL PROTECTED]
 Sent: Monday, October 18, 2004 10:54 AM
 Subject: Re: [obm-l] OBM 2004 - NIVEL 3 (2)


  Tinha esquecido uma linha:
 
  445566
  001144
  001166
  001177
  445566
  445566
  2233885588
  223388
  223399
  445566
 
  - Original Message -
  From: Paulo Rodrigues [EMAIL PROTECTED]
  To: [EMAIL PROTECTED]
  Sent: Monday, October 18, 2004 9:25 AM
  Subject: Re: [obm-l] OBM 2004 - NIVEL 3
 
 
  Acho que o exemplo abaixo está certo. Confira por favor
 
  445566
  001144
  001166
  001177
  445566
  445566
  2233885588
  223388
  445566
  - Original Message -
  From: Igor Oliveira [EMAIL PROTECTED]
  To: [EMAIL PROTECTED]
  Sent: Monday, October 18, 2004 8:55 AM
  Subject: [obm-l] OBM 2004 - NIVEL 3
 
 
E aí pessoal, como vocês foram na 3ª FASE (nível 3) desse ano? Eu
achei
 a
  prova
  desse ano muito mais dificil do que a do ano passado Acho que não
  consegui
  fazer nenhuma questão inteira...
 
Uma pergunta...: qual é a resposta da 4ª Questão, aquela do tabuleiro?
 Eh
  n=4 ou
  n=5? Não consegui achar um tabuleiro onde n=4. Se a resposta for n=4
 mesmo,
  me
  digam aí como preencher o tabuleiro.
 
 
Igor
 
 
 
=
  Instruções para entrar na lista, sair da lista e usar a lista em
  http://www.mat.puc-rio.br/~nicolau/olimp/obm-l.html
 
=
 
 
  ---
  Outgoing mail is certified Virus Free.
  Checked by AVG anti-virus system (http://www.grisoft.com).
  Version: 6.0.778 / Virus Database: 525 - Release Date: 15/10/2004
 
 
=
  Instruções para entrar na lista, sair da lista e usar a lista em
  http://www.mat.puc-rio.br/~nicolau/olimp/obm-l.html
 
=
 
 
=
  Instruções para entrar na lista, sair da lista e usar a lista em
  http://www.mat.puc-rio.br/~nicolau/olimp/obm-l.html
 
=


 ---
 Outgoing mail is certified Virus Free.
 Checked by AVG anti-virus system (http://www.grisoft.com).
 Version: 6.0.778 / Virus Database: 525 - Release Date: 15/10/2004

 =
 Instruções para entrar na lista, sair da lista e usar a lista em
 http://www.mat.puc-rio.br/~nicolau/olimp/obm-l.html
 =

 =
 Instruções para entrar na lista, sair da lista e usar a lista em
 http://www.mat.puc-rio.br/~nicolau/olimp/obm-l.html
 =


---
Outgoing mail is certified Virus Free.
Checked by AVG anti-virus system (http://www.grisoft.com).
Version: 6.0.778 / Virus Database: 525 - Release Date: 15/10/2004

=
Instruções para entrar na lista, sair da lista e usar a lista em
http://www.mat.puc-rio.br/~nicolau/olimp/obm-l.html
=


[obm-l] Re: [obm-l] Re: [obm-l] Combinatória

2004-09-28 Por tôpico Igor Castro
Title: Re: [obm-l] Combinatória



não entendi os passos:
"onde o coeficiente de t^n eh n+1" pq?
"onde o coeficiente de t^n 
eh(n+1)(n+2)(n+3)/6" pq?
[]´s
Igor

  - Original Message - 
  From: 
  Marcio Cohen 
  To: [EMAIL PROTECTED] 
  Sent: Tuesday, September 28, 2004 2:00 
  PM
  Subject: [obm-l] Re: [obm-l] 
  Combinatória
  
   Fui tentar fazer essa conta na 
  marra pra ver como ficava.. 
  (t^10 - 1)^4 / (t-1)^4 = (t^10-1)^4 * 
  (1+t+t^2+...)^4= (t^40 - 4t^30 + 6t^20 - 4t^10 + 1) * 
  (1+t+t^2+...)^4
  Agora, 
  (1+t+t^2+t^3+t^4+t^5+t^6+t^7...)^4 = (1+2t 
  +3t^2+4t^3 + 5t^4 + 6t^5 + 7t^6 + 8t^7+...)^2, onde o coeficiente de t^n eh 
  n+1,
  = 1+4t+10t^2+20t^3+35t^4+56t^5+..., ondeo 
  coeficiente de t^n eh(n+1)(n+2)(n+3)/6
  
   Dessa forma, a resposta eh 6*[t^7] 
  -4*[t^17] + [t^27] = 8*9*10 - 4*3*19*20 + 28*29*5 = 220
  
  Concordo plenamente que eh mto 
  mais importante aprender porque isso está certo do que fazer a conta.. Eh soh 
  pq eu fiquei curioso pra ver se era mto chato fazer. Abraços,
  Marcio
  
  
  
  
- Original Message - 
From: 
Claudio Buffara 
To: [EMAIL PROTECTED] 
Sent: Tuesday, September 28, 2004 9:35 
AM
Subject: Re: [obm-l] Combinatória
Qual o coeficiente de t^27 no desenvolvimento de:(1 + t + 
t^2 + t^3 + t^4 + t^5 + t^6 + t^7 + t^8 + t^9)^4 ?Resposta (usando 
PARI-GP): 220.Minha pergunta pra voce: Por que isso tah 
certo?[]s,Claudio. on 28.09.04 02:45, [EMAIL PROTECTED] at [EMAIL PROTECTED] wrote:

  
Resolva x + y + w + z = 27 sendo que o 
  maior valor que as incógnitas podem assumir seja 9, ou seja, 0 
  = x, y, w, z = 9 

  ---Outgoing mail is certified Virus 
Free.Checked by AVG anti-virus system (http://www.grisoft.com).Version: 
6.0.771 / Virus Database: 518 - Release Date: 
28/9/2004


[obm-l] Re: [obm-l] Re: [obm-l] RE: [obm-l] Re: [obm-l] Re:[obm-l] Questão 5 - OBM

2004-09-12 Por tôpico Igor Castro
Acho que não né.. se não tbm esqueci.. 2^13.11^0 é 2^13(e 2=/=13)... Deviam
especificar melhor isso..
[]´s
Igor Castro
- Original Message -
From: Maurizio [EMAIL PROTECTED]
To: [EMAIL PROTECTED]
Sent: Sunday, September 12, 2004 5:06 PM
Subject: [obm-l] Re: [obm-l] RE: [obm-l] Re: [obm-l] Re:[obm-l] Questão 5 -
OBM


 Olá

 Valia considerar números como 2^13.11^0?
 Pq se valer, eu esqueci de alguns numeros...

 Abraços
 Maurizio Casalaspro


 Daniel Regufe escreveu:

  Eh Igor ... Eu tb fiz isso na prova ... fui no braço ... Achei 7
  algarismos
 
  Oq eu quero saber eh se tem uma solução mais bonita ...
 
  Qual foi a sua solução pra questão 4?
 
  Tentem ae ...
  Problema 4 - Determine todas as soluções da equação n*(2)^(n-1) + 1 =
  m^2, com n e m naturais.
 
  []`
  Daniel Regufe
 
 
 
  Fala Luiz, a questão era a seguinte:
  Dizemos que um número inteiro é sinistro se a soma de seus fatores
  primos é
  igual 'a soma dos expoentes de sua decomposição em fatores primos.
  Encontre
  todos os números sinistros de quatro algarismos.
  Essa eu fiz, não vo escrever a solução toda... mas vo dar a idéia...
  (se vc for tentar pare de ler aqui)
  .
  .
  .
  .
  .
  .
  .
  .
  .
  .
  .
  .
  .
  Seguinte.. primeiro prove que que não pode haver fator primo 11 ou
maior
  nessa decomposição...
  supondo que houvesse.. o menor sinistro que satisfaz isso é N=
  (2^12).11 que
  é maior que ... assim vc prova que só tem 2,3,5 e 7 nessa
  decomposição... aí vc vai fazendo os casos na mão agora(não são
  muitos que
  servem) e também é bom fazer de uma maneira esperta.. começando do
  menor ou
  do maior sinistro pra poder já eliminar muitos casos...
  Acho que é braço mesmo... minha solução levou uma folha e meia.. deve
  ter
  uma solução mais bonita..  mas acho que achei todos os casos de
  primos com 4
  alg..(não são muitos)..
  []´s
  Igor Castro
  - Original Message -
  From: Luiz H. Barbosa [EMAIL PROTECTED]
  To: obm-l [EMAIL PROTECTED]
  Sent: Sunday, September 12, 2004 3:12 PM
  Subject: [obm-l] Re:[obm-l] Questão 5 - OBM
 
 
   Alo lista ... Queria ver uma resolução da questão 5 da
   OBM nivel 3 desse ano! ( A dos numeros sinistros ).. E
   se puderem a resolução da 4 tb!
  
   []`
   Daniel Regufe
  
   
   Não era melhor mandar as questões para a lista ?
   Não foi todo mundo que fez a prova aqui !!!
   []'s
   Luiz H. Barbosa
  
  
 
__
 
   Acabe com aquelas janelinhas que pulam na sua tela.
   AntiPop-up UOL - É grátis!
   http://antipopup.uol.com.br/
  
  
  
  
 
=
 
   Instruções para entrar na lista, sair da lista e usar a lista em
   http://www.mat.puc-rio.br/~nicolau/olimp/obm-l.html
  
 
=
 
  
 
 
 
=
 
  Instruções para entrar na lista, sair da lista e usar a lista em
  http://www.mat.puc-rio.br/~nicolau/olimp/obm-l.html
 
=
 
 
 
  _
  MSN Hotmail, o maior webmail do Brasil.  http://www.hotmail.com
 
 
=
  Instruções para entrar na lista, sair da lista e usar a lista em
  http://www.mat.puc-rio.br/~nicolau/olimp/obm-l.html
 
=
 
 

 =
 Instruções para entrar na lista, sair da lista e usar a lista em
 http://www.mat.puc-rio.br/~nicolau/olimp/obm-l.html
 =



=
Instruções para entrar na lista, sair da lista e usar a lista em
http://www.mat.puc-rio.br/~nicolau/olimp/obm-l.html
=


Re: [obm-l] geometria plana

2004-09-10 Por tôpico Igor Castro
Falso... isso vale sempre só quando o quadrilatero é inscritivel...
[]´s
Igor Castro

- Original Message - 
From: seanjr [EMAIL PROTECTED]
To: obm-l [EMAIL PROTECTED]
Sent: Friday, September 10, 2004 10:21 PM
Subject: [obm-l] geometria plana


 Julgue o item e justifique sua resposta.

 i) Dado um quadrilátero qualquer o produto da medida de suas
 diagonais é igual a soma dos produtos das medidas de seus
 lados não-adjacentes.

 qualquer ajuda é bem-vinda.

 __
 Acabe com aquelas janelinhas que pulam na sua tela.
 AntiPop-up UOL - É grátis!
 http://antipopup.uol.com.br/



 =
 Instruções para entrar na lista, sair da lista e usar a lista em
 http://www.mat.puc-rio.br/~nicolau/olimp/obm-l.html
 =

=
Instruções para entrar na lista, sair da lista e usar a lista em
http://www.mat.puc-rio.br/~nicolau/olimp/obm-l.html
=


[obm-l] Re: [obm-l] Combinatória cíclica

2004-09-03 Por tôpico Igor Castro





  - Original Message - 
  From: 
  claudio.buffara 
  To: obm-l 
  Sent: Saturday, August 28, 2004 3:59 
  PM
  Subject: [obm-l] Re:[obm-l] mais uma de 
  combinatória então
  
  
  Primeiroas mulheres se sentam: 11!
  Uma vez sentadas, as mulheres determinam 12 lugares, os quais devem ser 
  ocupados pelos 7 homens: 12!/(12-7)! = 12!/5!
  
  Total = 11!*12!/5! = 11!*11!/10
  
  []s,
  Claudio.
  
  
  


  De:
  [EMAIL PROTECTED]
  
  


  Para:
  [EMAIL PROTECTED]
  
  


  Cópia:
  
  
  


  Data:
  Sat, 28 Aug 2004 15:02:50 
-0300 (ART)
  
  


  Assunto:
  [obm-l] mais uma de 
combinatória então
  
  


  
  
   Aqui está mais um probleminha de combinatória.
   Tem a ver com permutação circular também.
   Não deve ser tão difícil, acho que não estou pensando do jeito 
  certo.
   Bom, aí está: 
   
   De quantas maneiras 7 homens e 12 mulheres podem sentar-se ao redor 
  de uma mesa redonda de forma que 2 homens não sentem juntos?
   
   
   
   Resp:
   
   (11!) * (11!) / 10
   
   Abraços,
   André Silveira Ramos
  
  
  Yahoo! 
  Acesso Grátis - navegue de graça com conexão de qualidade! 



Re: [obm-l] ITA

2004-09-02 Por tôpico Igor Castro
De onde você tirou essa média ponderada???


- Original Message - 
From: Fernando Aires [EMAIL PROTECTED]
To: [EMAIL PROTECTED]
Sent: Thursday, September 02, 2004 4:38 PM
Subject: Re: [obm-l] ITA


 Charles,

Eles têm um critério esquisito:
Você tem que acertar 40% de cada prova (Português - P, Inglês - I,
 Matemática - M, Física - F e Química - Q), e ter pelo menos 50% de
 aproveitamento sob uma média ponderada: (M.1,2+F.1+Q.1+P.0,8+I.0)/4.
 Ou seja, Inglês não conta para esta média.
Dado isso, eles classificam os alunos por essa média ponderada, e
 chamam os primeiros colocados...

Ao menos era assim quando eu prestei...

 Beijos,

 -- 
 --
 Fernando Aires
 [EMAIL PROTECTED]
 Em tudo Amar e Servir
 --

 - Original Message -
 From: Charles Quevedo [EMAIL PROTECTED]
 Date: Thu, 2 Sep 2004 14:06:54 -0300 (ART)
 Subject: [obm-l] ITA
 To: [EMAIL PROTECTED]


 Alguem da lista sabe me informar qual a media de acertos para passar no
ITA.

 =
 Instruções para entrar na lista, sair da lista e usar a lista em
 http://www.mat.puc-rio.br/~nicolau/olimp/obm-l.html
 =

=
Instruções para entrar na lista, sair da lista e usar a lista em
http://www.mat.puc-rio.br/~nicolau/olimp/obm-l.html
=


Re: [obm-l] probleminha, estudo de sinal

2004-09-02 Por tôpico Igor Castro
Depois de garantir que as duas raizes existem(delta0) faz Produto0 e Soma
0. E faz a interseção de tudo...
[]´s
Igor Castro

- Original Message - 
From: Bruno França dos Reis [EMAIL PROTECTED]
To: OBM [EMAIL PROTECTED]
Sent: Thursday, September 02, 2004 11:03 PM
Subject: [obm-l] probleminha, estudo de sinal


 Ola

 Estava resolvendo um problema que envolve estudo de sinal. Minha
 resolucao ficou grande, nao sei se teria alguma forma de ficar mais
 simples. Alguem poderia verificar?

 f(x)=ax^2 + (a-1)x + (a-1)

 Determine todos os a E |R tais que f(x)=0 tenha duas raizes negativas.

 O que eu fiz foi aplicar um bhaskara. Fiz entao que
 Delta  0, cheguei em -1/3  a  1

 verifiquei que 1-a+sqrt(delta) é sempre positivo, entao teria que ter
 2a (que é o denominador) negativo sempre.
 (*) Entao -1/3a0.

 Fui entao verificar se teria que restringir ainda mais o intervalo:
 1-a-sqrt(delta)0 = 1-asqrt(delta) 1-2a + a^2 |delta|
 1) 1-2a + a^2  delta = a0 ou a1 = descartei a1 pela C.E.
 2) 1-2a+a^2  -delta = a1 = redundante por causa de (*)

 Entao, a E ]-1/3, 0[

 Acho que a resposta certa é essa (uma amiga me passou o problema, e
 acho que me disse isso ae mesmo qd conferiu no gabarito).

 É necessario fazer toda essa analise? Ou teria algum lugar para parar
 antes? A analise que fiz do modulo está correta?


 Obrigado!
 abraco


 -- 
 Bruno França dos Reis
 email: bfreis - gmail.com
 gpg-key: http://planeta.terra.com.br/informatica/brunoreis/brunoreis.key
 icq: 12626000

 =
 Instruções para entrar na lista, sair da lista e usar a lista em
 http://www.mat.puc-rio.br/~nicolau/olimp/obm-l.html
 =

=
Instruções para entrar na lista, sair da lista e usar a lista em
http://www.mat.puc-rio.br/~nicolau/olimp/obm-l.html
=


[obm-l] Re: [obm-l] Desigualdade de Médias

2004-09-02 Por tôpico Igor Castro



Tem uma demonstração legal usando gráfico eo 
fato de o centro de massa de um poligono convexo estar no seu interior(acho que 
na verdade isso também envolve funções convexas se vc quiser ser mais 
detalhista)...
Faça o gráfico da função ln(x)
pegue n pontossobre essegráfico 
(ln(a1), ln(a2)  ln (an) )
Desenhe o poligono correspondente a esses n 
pontos... ele ficará "abaixo" da curva do ln.. seu centro de massa( de 
coordenada y = (ln(a1), ln(a2)  ln(an))/n)) é um ponto dentro desse 
poligono...
Agora pegue o ponto 
ln((a1+a2+a3+...an)/n)sobre o 
gráfico..
Esse ponto está acima do poligono(pois está na 
curva), logo, acima do seu centro de massa..
então...
ln((a1+a2+a3+...an)/n) = (ln(a1), ln(a2) 
 ln (an))/n)
ln (a1+a2+...an)/n)  = ln ((a1.a2.a3..an)^(1/n) 
e como ln é crescente...
(a1+a2+...an)/n) = (a1.a2.a3..an)^(1/n) .:. Ma 
= Mg
tem outras demonstrações também... acho que vc deve 
achar na internet(caso alguem não poste)
[]´s
Igor



  - Original Message - 
  From: 
  Douglas Ribeiro Silva 
  To: [EMAIL PROTECTED] 
  Sent: Thursday, September 02, 2004 11:51 
  PM
  Subject: [obm-l] Desigualdade de 
  Médias
  
  
  Olá 
  pessoal.
  
  Ultimamente eu me deparei com uma 
  questão de média aritmética x geométrica e fiquei curioso pra saber a 
  generalização da desigualdade da mesma. Dei uma olhada no arquivo da lista e 
  achei esse link onde o Morgado mostrou: http://www.mat.puc-rio.br/~nicolau/olimp/obm-l.27/msg00188.html
  Entendi o modo como foi feito para 
  quantidade de números potencias de 2, mas não compreendi os passos utilizados 
  na indução. Alguém poderia detalhar melhor os passos da demonstração e/ou 
  mandar outras demonstrações dessa generalização?
  
  Um abraço, 
  Douglas


Re: [obm-l] ITA

2004-09-02 Por tôpico Igor Castro
Tá...
sempre ouvi dizer que era a média arit sem ingles...
Mas bem, como os organizadores do vest adoram esclarecer as coisas, vamos
continuar sem saber..
[]´s

- Original Message - 
From: Fernando Aires [EMAIL PROTECTED]
To: [EMAIL PROTECTED]
Sent: Friday, September 03, 2004 12:05 AM
Subject: Re: [obm-l] ITA


 Igor,

Quando eu fui conhecer o ITA, um dos professores lá me mostrou essa
conta...
Agora, não me pergunta o nome dele, porque eu só sei meu nome
 olhando no RG... :-)

 Beijos,

 -- 
 --
 Fernando Aires
 [EMAIL PROTECTED]
 Em tudo Amar e Servir
 --

 On Thu, 2 Sep 2004 22:30:09 -0300, Igor Castro [EMAIL PROTECTED]
wrote:
  De onde você tirou essa média ponderada???

 =
 Instruções para entrar na lista, sair da lista e usar a lista em
 http://www.mat.puc-rio.br/~nicolau/olimp/obm-l.html
 =

=
Instruções para entrar na lista, sair da lista e usar a lista em
http://www.mat.puc-rio.br/~nicolau/olimp/obm-l.html
=


[obm-l] Re: [obm-l] [obm-l] Polinomio - retificação

2004-07-18 Por tôpico Igor Castro
E a soma, é só das reais tbm? e o produto?
- Original Message - 
From: Daniel Regufe [EMAIL PROTECTED]
To: [EMAIL PROTECTED]
Sent: Sunday, July 18, 2004 12:03 PM
Subject: [obm-l] [obm-l] Polinomio - retificação


 To achando algo de errado nessa questão ... mas tentem fazer pra mim ...
 Seja p(x) = 16x^5 - 78x^4 + ... + mx - 5 um polinomio de coeficientes
reais
 tal que a equação p(x) = 0 admite mais do que uma raiz real e ainda, a +
bi
 é uma raiz complexa desta equação com a e b diferentes de zero.
Sabendo-se
 que 1/a é a razão da progressão geométrica formada pelas raízes
 reais de p(x) = 0 e que a soma destas raízes vale 7/8 enquanto que
o
 produto é 1/6, o valor de m é : ...
 
 Desculpe ... nao tinha colocado q a progressão era soh das raizes reais ..

 []´s
 Regufe
 
 _
 MSN Hotmail, o maior webmail do Brasil.  http://www.hotmail.com
 
 =
 Instruções para entrar na lista, sair da lista e usar a lista em
 http://www.mat.puc-rio.br/~nicolau/olimp/obm-l.html
 =

 _
 MSN Hotmail, o maior webmail do Brasil.  http://www.hotmail.com

 =
 Instruções para entrar na lista, sair da lista e usar a lista em
 http://www.mat.puc-rio.br/~nicolau/olimp/obm-l.html
 =


=
Instruções para entrar na lista, sair da lista e usar a lista em
http://www.mat.puc-rio.br/~nicolau/olimp/obm-l.html
=


[obm-l] Re: [obm-l] RE: [obm-l] Re: [obm-l] [obm-l] Polinomio - retificação

2004-07-18 Por tôpico Igor Castro
Se tem mais de uma real, então tem 3(pois as complexas aparecem aos pares,
nesse caso a+bi e a-bi).
sendo r1 uma raiz real, podemos ter as outras reais com r1/a e r1/(a^2).
As 3 reais: r1, r1/a e r1/a^2
soma: r1(1 + 1/a + 1/a^2) = 7/8
produto: r1^3/a^3 = 1/6
soma das 5 raizes: 7/8 + a+bi + a-bi = 78/16  - a = 2
logo, pelo produto das 3 reais r1^3 = 4/3 -  r^1 = sqrt3(4/3)
Confirando na soma das 3 reais - sqrt3(4/3)(1 + 1/2 + 1/4) = 7/8 (absurdo)
Se não me confundi, tem algo estranho mesmo... Talvez o a da raiz complexa
não seja o mesmo a da razão da pg..
[]´s
Igor

- Original Message -
From: Daniel Regufe [EMAIL PROTECTED]
To: [EMAIL PROTECTED]
Sent: Sunday, July 18, 2004 2:48 PM
Subject: [obm-l] RE: [obm-l] Re: [obm-l] [obm-l] Polinomio - retificação


 sim ...a soma e o produto se refere as raizes reais ... tenta ae pra mim

 []`s
 Regufe


 From: Igor Castro [EMAIL PROTECTED]
 Reply-To: [EMAIL PROTECTED]
 To: [EMAIL PROTECTED]
 Subject: [obm-l] Re: [obm-l] [obm-l] Polinomio - retificação
 Date: Sun, 18 Jul 2004 14:35:02 -0300
 
 E a soma, é só das reais tbm? e o produto?
 - Original Message -
 From: Daniel Regufe [EMAIL PROTECTED]
 To: [EMAIL PROTECTED]
 Sent: Sunday, July 18, 2004 12:03 PM
 Subject: [obm-l] [obm-l] Polinomio - retificação
 
 
   To achando algo de errado nessa questão ... mas tentem fazer pra mim
 ...
   Seja p(x) = 16x^5 - 78x^4 + ... + mx - 5 um polinomio de coeficientes
 reais
   tal que a equação p(x) = 0 admite mais do que uma raiz real e ainda,
a
 +
 bi
   é uma raiz complexa desta equação com a e b diferentes de zero.
 Sabendo-se
   que 1/a é a razão da progressão geométrica formada pelas raízes
   reais de p(x) = 0 e que a soma destas raízes vale 7/8 enquanto
 que
 o
   produto é 1/6, o valor de m é : ...
   
   Desculpe ... nao tinha colocado q a progressão era soh das raizes
reais
 ..
  
   []´s
   Regufe
   
   _
   MSN Hotmail, o maior webmail do Brasil.  http://www.hotmail.com
   
  
 
=
   Instruções para entrar na lista, sair da lista e usar a lista em
   http://www.mat.puc-rio.br/~nicolau/olimp/obm-l.html
  
 
=
  
   _
   MSN Hotmail, o maior webmail do Brasil.  http://www.hotmail.com
  
  
 =
   Instruções para entrar na lista, sair da lista e usar a lista em
   http://www.mat.puc-rio.br/~nicolau/olimp/obm-l.html
  
 =
 
 
 =
 Instruções para entrar na lista, sair da lista e usar a lista em
 http://www.mat.puc-rio.br/~nicolau/olimp/obm-l.html
 =

 _
 MSN Messenger: converse com os seus amigos online.
 http://messenger.msn.com.br

 =
 Instruções para entrar na lista, sair da lista e usar a lista em
 http://www.mat.puc-rio.br/~nicolau/olimp/obm-l.html
 =



=
Instruções para entrar na lista, sair da lista e usar a lista em
http://www.mat.puc-rio.br/~nicolau/olimp/obm-l.html
=


Re: [obm-l] Postulados ...

2004-07-18 Por tôpico Igor Castro
Acho que é só uma questão de definição... Não sei qual é a certa(imagino que
seja a segunda hipótese porque perpendiculares já traria consigo o fato de
serem coplanares e reversas são não coplanares).
[]´s
Igor

- Original Message -
From: Daniel Regufe [EMAIL PROTECTED]
To: [EMAIL PROTECTED]
Sent: Sunday, July 18, 2004 5:43 PM
Subject: [obm-l] Postulados ...


 Queria a opinião da galera da lista sobre o seguinte detalhe  ...

 Vcs acham q retas perpendiculares eh um caso de retas ortogonais ou retas
 ortogonais devem ser necessariamente reversas???

 []`s
 Regufe

 _
 MSN Messenger: converse com os seus amigos online.
 http://messenger.msn.com.br

 =
 Instruções para entrar na lista, sair da lista e usar a lista em
 http://www.mat.puc-rio.br/~nicolau/olimp/obm-l.html
 =



=
Instruções para entrar na lista, sair da lista e usar a lista em
http://www.mat.puc-rio.br/~nicolau/olimp/obm-l.html
=


Re: [obm-l] Escola Naval 2004

2004-07-18 Por tôpico Igor Castro



(sem as setinhas de vetor):
U = (2,1,-3)
P= (3,-1,0)
Seja V=(a,b,c) e W=(d,e,f)
W e V são perpendiculares -  ad + be + cf = 0 
(produto escalar=0)
2 = a+d - 4 = a^2 + 2ad + d^2
1 
= b^2 + 2be + e^2
 
9 = c^2 +2cf + f^2
- 14 =a^2 + b^2 + c^2 + d^2 + e^2 + f^2+ 2(ad + be + cf) = a^2 + b^2 + 
c^2 + d^2 + e^2
V - W = (a - d, b - e, c - f) -  |V - W| = 
sqrt2(a^2 + b^2 + c^2 + d^2 + e^2 + f^2 - 2(ad + be + cf) =
sqrt2(a^2 + b^2 + c^2 + d^2 + e^2 + f^2) = 
sqrt2(14) (letra B)
Avisem se tiver algo errado..
[]´s
Igor

  - Original Message - 
  From: 
  João Vitor 
  
  To: [EMAIL PROTECTED] 
  Sent: Sunday, July 18, 2004 8:12 PM
  Subject: Re: [obm-l] Escola Naval 
  2004
  
  Essa é de Vetores
  
   
  -- - 
  - 
  -- 
  - 
  - 
  --- 
  -
  Sabendo q: U = 2i + 
  j - 3k ; U = V + W onde V é 
  paralelo a P = 3i - 
  j e W é
  
   
  - 
  - -
  perpendicular a P ; Podemos Afirmar q 
  |V - W| é:
  
  A) Sqrt(19)/2
  
  B) Sqrt(14)
  
  C) Sqrt(27)/4
  
  D) Sqrt(20)
  
  E) Sqrt(53)/2
  
  Essa caiu ano passado na Escola 
  Naval!
  
  
  
  João Vitor, Fortaleza - CE
  
- Original Message - 
From: 
Robÿe9rio Alves 
To: [EMAIL PROTECTED] 
Sent: Saturday, July 17, 2004 8:03 
PM
Subject: [obm-l] Probleminha legal, 
como resolver ?

Sabendo que um balaio de ovo foi dividido entre três pessoas. O 
primeiro ficou com a metade da quantidade de ovos mais meio ovo. O segundo 
ficou com a metade do que sobrou mais um muio. Por conseguinte, o último com 
a metade do que sobrou mais um meio. Pergunta - se 
a) Quantos ovos ( inteiros ) há no balaio ?
b) Quantos ovos ficou a primeira pessoa ?
c) Quantos ovos ficoua segunda ?
d) Quantos ovos ficoua terceira ?
__Do You 
Yahoo!?Tired of spam? Yahoo! Mail has the best spam protection around 
http://mail.yahoo.com 


[obm-l] Resultado IMO

2004-07-16 Por tôpico Igor Castro



Não sei se é oficial:
http://www.mathlinks.ro/viewtopic.php?t=14187
Parece que o brasil ficou em 21º. Na frente de 
várias nações mais "importantes", como a alemanha, itália e frança. 

Parabens ao time brasileiro
[]´s
Igor


Re: [obm-l] log-poli

2004-07-15 Por tôpico Igor Castro



1/bc + 1/ac + 1/ab = (a + b + c)/abc = Soma das 
raízes / Produto
Pelas relações de girard em 2x^3 - 
30x^2+15x - 3x = 0 - soma das 3 raízes= -(-30)/2 e produto 
= -(-3)/2
soma /produto = 10
e o log pedido é -1 pois (1/10)^(-1) = 
10
[]´s
Igor


  - Original Message - 
  From: 
  [EMAIL PROTECTED] 
  
  To: [EMAIL PROTECTED] 
  Cc: [EMAIL PROTECTED] 
  Sent: Thursday, July 15, 2004 9:18 
  AM
  Subject: [obm-l] log-poli
  Questão de logarítimo com 
  polinômioanexoabraçosJunior 
  
  

  
  
  


[obm-l] Re: [obm-l] Geometria e Aritmética

2004-07-07 Por tôpico Igor Castro
*1º - Reta: mx + py + q = 0
Distancia de A(Xa,Ya) à r -  |mXa + pYa + q| / (m^2 + p^2)^1/2
 // B|mXb + pYb + q| / (m^2 +
p^2)^1/2
//  C|mXc + pYc + q| / (m^2 +
p^2)^1/2
Soma das distancias -  | m(Xa + Xb + Xc) + p(Ya + Yb + Yc) + 3q| / (m^2 +
p^2)^1/2
Distancias ao baricentro(Xa+Xb+Xc/3, Ya + Yb + Yc/3) -  | m(Xa + Xb + Xc)/3
+ p(Ya + Yb + Yc)/3 + q| / (m^2 + p^2)^1/2 = 1/3 da soma das distancias.
Acho q eh isso se ñ me enrolei nos modulos..

*2º use o 1º

[]´s
Igor


- Original Message - 
From: victor machado [EMAIL PROTECTED]
To: [EMAIL PROTECTED]
Sent: Wednesday, July 07, 2004 9:19 PM
Subject: [obm-l] Geometria e Aritmética


 Amigos da OBM-l, gostaria de saber a solução dos seguintes exercícios:

 - ABC é um triângulo cujo baricentro é G e R é uma reta externa ao
triângulo.
 Demonstrar que a soma das distâncias dos vértices A, B e C do
 triângulo à reta R é igual ao triplo da distância de G a esta mesma
 reta.

 - As distâncias dos três vértices de um triângulo a uma reta medem 7m,
 9, e 14m, respectivamente. Calcular a distância do baricentro do
 triângulo à mesma reta.

 - Uma urna contém 100 bolas pretas e 100 bolas brancas. De forma
 repetitiva, retiramos três bolas de urna e as substituírmos por outras
 existentes num monte de bolas fora da urna, da seguintes forma:

 BOLAS REMOVIDAS / SUBSTITUÍDAS POR
 3 pretas / 1 preta
 2 pretas e 1 branca / 1 preta e 1 branca
 1 preta e 2 brancas / 2 brancas
 3 brancas / 1 preta e 1 branca

 Qual dos conjuntos de bolas abaixo pode ser o conteúdo da urna após
 isto ser feito muitas vezes ?

 A) 2 bolas pretas
 B) 2 bolas brancas
 C) 1 bola preta
 D) 1 bola preta e 1 bola branca
 E) 1 bola branca

 - Uma raposa está adiantada 60 pulos seus sobre um cão que a persegue.
 Enquanto que a raposa dá 10 pulos, o cão dá 8; 3 pulos do cão valem 5
 pulos da raposa. Quantos pulos dará o cão para alcançar a raposa ?

 - Duas estradas de iguais dimensões começam simultâneamente a ser
 construídas por 15 operários cada uma delas. Mas, exclusivamente
 devido a dificuldades no terreno, percebe-se que enquanto uma turma
 avançous 2/3 na sua obra, a outra avançou 4/5 da sua. O número de
 operários que devemos tirar de uma e por na outra para que as duas
 obras fiquem prontas ao mesmo tempo é ?

 Eu sei que são muitas, mas agradeço mesmo assim.
 muito obrigado,
 victor.

 =
 Instruções para entrar na lista, sair da lista e usar a lista em
 http://www.mat.puc-rio.br/~nicolau/olimp/obm-l.html
 =

=
Instruções para entrar na lista, sair da lista e usar a lista em
http://www.mat.puc-rio.br/~nicolau/olimp/obm-l.html
=


Re: [obm-l] Geometria dos Complexos

2004-06-17 Por tôpico Igor Castro
Que tal no site da obm?
Vá em semana olímpica e busque o material da aula do prof. Marcio Cohen
sobre isso.. É muito bom.
[]´s
Igor (Castro)

- Original Message - 
From: Igor Oliveira [EMAIL PROTECTED]
To: [EMAIL PROTECTED]
Sent: Thursday, June 17, 2004 12:26 AM
Subject: [obm-l] Geometria dos Complexos


   Gostaria que vocês me indicassem sites onde posso encontrar teoria,
aplicações e
 exercícios resolvidos  sobre NÚMEROS COMPLEXOS APLICADOS EM GEOMETRIA, ou
 GEOMETRIA COM NÚMEROS COMPLEXOS. Acho que é a mesma coisa. Obrigado

   Igor
 =
 Instruções para entrar na lista, sair da lista e usar a lista em
 http://www.mat.puc-rio.br/~nicolau/olimp/obm-l.html
 =

=
Instruções para entrar na lista, sair da lista e usar a lista em
http://www.mat.puc-rio.br/~nicolau/olimp/obm-l.html
=


Re: [obm-l] Problema envolvendo sistema linear

2004-06-15 Por tôpico Igor Castro
p - nº de peras
l - nº de laranjas
3l = p
sejam n amigos
5n = l
8n + 21 = p
daí, 8n + 21 = 3l = 15n -7n=21 - n=3(3 amigos) ; l = 5n - 15 laranjas
- Original Message - 
From: Maurizio [EMAIL PROTECTED]
To: [EMAIL PROTECTED]
Sent: Tuesday, June 15, 2004 7:16 PM
Subject: [obm-l] Problema envolvendo sistema linear


 Em uma cesta de frutas, há 3 vezes mais peras doq ue laranjas. Eu e meus
 amigos vamos dividir as frutas. Se cada um de nós receber 5 laranjas e 8
 peras, restarão 21 peras, e as laranjas serão todas distribuídas.
 Quantas laranjas há na cesta? Quantas pessoas somos?
 =
 Instruções para entrar na lista, sair da lista e usar a lista em
 http://www.mat.puc-rio.br/~nicolau/olimp/obm-l.html
 =

=
Instruções para entrar na lista, sair da lista e usar a lista em
http://www.mat.puc-rio.br/~nicolau/olimp/obm-l.html
=


Re: [obm-l] Soma dos Quadrados...

2004-06-13 Por tôpico Igor Castro



a^2 - b^2 = 21 - (a+b)(a-b)=3x7 .. se for 
nosnaturais podemos ter..(lembra que é só pra achar um valor 
possível...)
a+b=7
a-b=3
- a=5
- b=2
25+4=29 (a)
[]´s



  - Original Message - 
  From: 
  Fabio Contreiras 
  To: [EMAIL PROTECTED] 
  Sent: Sunday, June 13, 2004 12:06 
PM
  Subject: [obm-l] Soma dos 
  Quadrados...
  
  Ola galera!, qual seria um bizu maneiro pra 
  resolver essa questão ?
  1) A diferença entre os quadrados de dois numeros 
  naturais é 21. Um dos possiveis valores da soma dos quadrados desses dois 
  numeros é ?
  
  a ) 29
  b ) 97
  c) 132
  d ) 184
  e ) 252
  
  
  imaginei x^2 - y^2 = 21
  tentei desmembrar ( x + y ) ( x - y ) = 21 , mas 
  nao consegui relacionar com x^2 + y^2 ...
  
  Abraços!
  


Re: [obm-l] Problema 16 OBM - Nivel 3

2004-06-07 Por tôpico Igor Castro
Substitua x=1, também serve.
Aproveitando, alguém sabe quando sairá o gabarito? Normalmente sai na
segunda posterior à prova. Parece que dessa vez esqueceram
[]´s
Igor Castro

- Original Message -
From: Maurizio [EMAIL PROTECTED]
To: [EMAIL PROTECTED]
Sent: Monday, June 07, 2004 7:22 PM
Subject: [obm-l] Problema 16 OBM - Nivel 3


 Olá
 a questão 16 é assim:

 [x+2(x-1)^1/2]1/2+[x-2(x-1)^1/2]1/2=2

 Eu obtive essa resoluçãoi mas não está dando certo... Quem escrever
 alguma resolução ou indicar o erro da minha eu agradeço desde já

{[x+2(x-1)^1/2]1/2}^2+2{[x+2(x-1)^1/2]1/2.[x-2(x-1)^1/2]1/2]}+{[x-2(x-1)^1/2
]1/2}^2=4
 x+2[x-1]^1/2+2{x^2-2[x-1]^1/2}^1/2+x-2[x-1]^1/2=4
 2x+2[x^2-4(x-1)]^1/2=4
 x=2
 =
 Instruções para entrar na lista, sair da lista e usar a lista em
 http://www.mat.puc-rio.br/~nicolau/olimp/obm-l.html
 =



=
Instruções para entrar na lista, sair da lista e usar a lista em
http://www.mat.puc-rio.br/~nicolau/olimp/obm-l.html
=


Re: [obm-l] En:colegio naval

2004-05-24 Por tôpico Igor Castro
Bota o 13 em evidencia.. fica
13( 1/2*4 + 1/4*6 + 1/6*8 ... + 1/52*54)
agora veja que 1/(2*4) = (1/2 - 1/4) / 2
// // //1/(4*6) = (1/4 - 1/6)/2
 .. ...
...
 1/(48*50) = (1/48 - 1/50)/2
  1/(50*52) = (1/50 - 1/52)/2
somando tudo vc vai ver q os termos vão se cortar com excessão do 1/2 e
do -1/52
Logo
13(1/2 - 1/52)/2 = (13/2)*(25/52) mas 52 = 13.4, logo o valor é: 25/(2*4)=
25/8
[]´s
Igor Castro


- Original Message -
From: leandro-epcar [EMAIL PROTECTED]
To: obm-l [EMAIL PROTECTED]
Sent: Monday, May 24, 2004 10:22 AM
Subject: [obm-l] En:colegio naval


 -- Início da mensagem original ---

   De: quot;leandro-epcarquot; leandro-
 [EMAIL PROTECTED]
 Para: quot;obm-lquot; [EMAIL PROTECTED]
   Cc:
 Data: Mon, 24 May 2004 10:08:21 -0300
  Assunto: colegio naval

   Alguem poderia me dar uma ideia nesta questao ,nao
 consigo achar uma sequencia ,nem mesmo calcular, esta
 questao.
grato
 leandro



Colegio naval   1994

   Sabendo-se que a seguinte identidade  (AX + BY)/XY =
 A/Y + B/X é verdadeira para quaisquer números reais A,B,
 X0,Y0,
   o valor de 13/(2*4)+ 13 /(4*6)+ 13/ ( 6*8) +...+13/
 (50*52)

 (A)25/16
 (B)25/12
 (C)25/8
 (D)25/4
 (E)25/2


 
 __
 Acabe com aquelas janelinhas que pulam na sua tela.
 AntiPop-up UOL - É grátis!
 http://antipopup.uol.com.br/



 __
 Acabe com aquelas janelinhas que pulam na sua tela.
 AntiPop-up UOL - É grátis!
 http://antipopup.uol.com.br/



 =
 Instruções para entrar na lista, sair da lista e usar a lista em
 http://www.mat.puc-rio.br/~nicolau/olimp/obm-l.html

=r/~
nicolau/olimp/obm-l.html
 =



=
Instruções para entrar na lista, sair da lista e usar a lista em
http://www.mat.puc-rio.br/~nicolau/olimp/obm-l.html
=


Re: [obm-l] problemas de relogios

2004-04-09 Por tôpico Igor Castro



Veja o ângulo que cada ponteiro fará com a 
vertical.. assim.. supondo que tenha se passado h horas e m minutos
Angulo que o ponteiro das horasandou 
- alfa = 30h + 30(m/60) (o 30 eh pq cada hora equivale a 30 
graus)
angulo que o ponteiro dos minutos andou- 1 min 
= 6 graus, logo... beta = 6m
Aí, |Alfa - Beta| vai ser o anguloformado 
pelos ponteiros| 30h - 5,5m |
Mas veja que, por exemplo, 13 horas = 1 hora.. vc 
deve usar o 1 (a formula se baseia nos ponteirosde0 a 
11)
com isso vc faz os dois problemas mais facilmente.. 
ok?
no primeiro : |30.1 - 5,5.m| = 90 
- - ( 5,5m -30)= 90 (se vc naum invertesse ia ver que os minutos dariam 
negativos)

- 5,5m = 120 - m=120/5,5 = 21minutos e 
49,09 seg (21,81818181.. min= 21min + 0,818181x60 seg)

[]´s
Igor



 Original Message - 

  From: 
  [EMAIL PROTECTED] 
  
  To: [EMAIL PROTECTED] 
  Sent: Friday, April 09, 2004 2:18 
AM
  Subject: [obm-l] problemas de relogios 
  
  Ola 
  pessoal, Como resolver estes ? 1) Em que instante depois das 
  13:00 horas ficam os ponteiros de um relogio em angulo reto ? 2) A que 
  horas os ponteiros das horas e dos minutos de um relogio estarao no 
  prolongamento, um do outro, logo depois de estarem marcando precisamente 3 
  horas ? ps: Coloquei o gabarito la embaixo para quem quiser resolver e 
  comparar depois. gabarito 
  1) 13h 21min 49,1s 2) 3h 49min 60/11s 



Re: [obm-l] Exercicio Colegio Naval 2003 - Equacoes do segundo grau

2004-02-26 Por tôpico Igor Castro



Bem, fazendo uma analise rápida, a equação terá 
raízes racionais se raiz de deltaforracional

Delta = k^2 - 4.4.3 = x^2 -  (k+x)(k-x)= 4.4.3 
= 48
Bem, daih, pra cada A.B=48 que vc tiver.. vc tem um 
valor de k(note que assim x e k sempre serão racionais, soh resolver o 
sistema).. como o prb pede Quantos valores servem... é melhor vc deve olhar pro 
números de divisores de 48(tem uma formulado mto conhecida pra isso)ao 
invés de tentar achar todos e depois contar.. espero que dê certo essa 
contagem..
Espero que tenha ajudado um pouco...

Igor de Castro- www.cnaval.cjb.net



- Original Message - 

  From: 
  Victor 
  Machado 
  To: [EMAIL PROTECTED] 
  Sent: Thursday, February 26, 2004 8:12 
  PM
  Subject: [obm-l] Exercicio Colegio Naval 
  2003 - Equacoes do segundo grau 
  
  Olá amigos da Lista, queria lhes agradecer pelas 
  resolucoes enviadas.
  Mas gostaria de outra :
  
  (CN-2003) Dada a equação do 2º 
  grau na incógnita x : 4x^2 + Kx + 3 = 
  0. Quantos são os valores inteiros possíveis do parâmetro 
  K, tais que essa equação só admita raízes 
racionais?
  
  Falaram-me que o exercicio sairia facil pelo 
  teoremas das raizes racionais, mas nao o conheco... entao peco-lhes : poderiam 
  por a resolucao junto com uma pequena teoria sobre esse teorema ?
  
  Agradeco desde ja
  
  Victor


[obm-l] Re: [obm-l] Questões_IME_(ultra-foda!)

2003-10-01 Por tôpico Igor Castro



Dividir harmonicamente um segmento é "dividi-lo" 
internamente e externamente na mesma razão. Ex, AB=10, C está sobre AB tal que 
AC=6, D está depois de B tal que BD=20. Repare que AC/BC = AD/BD. Ou seja, C e D 
dividem AB harmonicamente numa certa razão(mais a fundo, vc pode provar 
que representam o pé das bissetrizes internas e externas e daí concluir mais 
coisas)
Não acho que os problemas do ime sejam mal 
elaborados. Na minha opinião, o que ocorre as vezes(poucas diante de 
todosos problemas)é que devido a complexidade dos problemas e a 
necessidade de adaptá-los a uma prova de concurso de nível médio pode deixar o 
enunciado um pouco estranho ou confuso. Masisso realmente ocorre poucas 
vezes.
[]´s
Igor Castro


- Original Message - 

  From: 
  Roberto Gomes 
  To: [EMAIL PROTECTED] 
  Sent: Wednesday, October 01, 2003 8:34 
  AM
  Subject: Re: [obm-l] 
  Questões_IME_(ultra-foda!)
  
  Não concordo com vc, pelo contrário acho as provas do IME muito bem 
  elaboradas, não vejo nada de confuso. sobre divisão harmonica e questões com 
  essa de geometria vc poderá encontra no livro Geometria II do Morgado que por 
  sinal, para mim, é uns dos melhores livros de geometria que eu conheço.
  
  Roberto GomesAlexandre Daibert [EMAIL PROTECTED] 
  wrote:
  Caros 
colegas, gostaria da ajuda dos senhores, por obséquio, se não for 
incomodar muito, para a resolução dos seguintes problemas de 
vestibulares do IME:(IME 96)Dados os trinômios de segundo 
grau:y = ax^2 + bx + c (I)y = a´x^2 + b´x + c´ (II)Cosidere, 
sobre o eixo Ox, os pontos cujas abscissas são as raízes do trinômio (I) 
e A´B´ os pontos cujas abscissas são raízes do trinômio (II).Determine a 
relação que deve existir entre os coeficientes a, b, c, a´, b´, c´, de 
modo que A´B´divida o segmento AB harmonicamente.obs1: O que 
significa esta divisão harmônica? As extremidades podem ser iguais? ou 
seria a divisão do segmento em 3? Como divido um segmento em 3 
harmonicamente?(IME 96)Determine os números naturais n para os 
quais existem poliedros convexos de n arestas.obs2: essa eu até 
fiz, mas gostar! ia de conferir a resposta.(IME 93)Num triângulo 
ABC traçamos a altura AH e do pé H desta altura construímos as 
perpendiculares HD e HE sobre os lados AB e AC; seja P o ponto da 
interseção de DE com BC. Construindo as alturas relativas aos vértices B 
e C determina-se também, de modo análogo Q e R sobre os lados CA, AB. 
Demonstre que os pontos P, Q, R são colineares.obs3: Esta questão 
tem uma figura, q eu considerei desnecessária. Caso alguém não tenha 
entendido me diga q eu faço a figura e mando pra lista.Gostaria de 
aproveitar a oportunidade em q estou abrindo a discussão destes 
problemas do IME para expressar a minha indignação sobre alguns 
problemas deste vestibular. Por anos temos visto que o IME não se cansa 
de colocar questões mal-elaboradas em seu vestibular, no sentido de ter 
interpretações ambíguas, não só na prova de matemática. Mesmo o aluno 
mais bem preparado fica confuso frente a algumas questões, que s! ão 
mal-colocadas realmente. Fico me perguntando qual o objetivo dos 
professores ao colocar questões confusas no vestibular. Selecionar os 
melhores candidatos, provavelmente não é, pois as vezes um bom candidato 
pode ser eliminado porque não soube interpretar uma questão confusa. 
Gostaria de saber se esta opinião é só minha, ou se mais algum colega da 
lista compartilha do mesmo sentimento em relação ao vestibular do 
IME.Alexandre 
D.=Instruções 
para entrar na lista, sair da lista e usar a lista 
emhttp://www.mat.puc-rio.br/~nicolau/olimp/obm-l.html=
  
  
  Yahoo! 
  Mail - o melhor webmail do Brasil. Saiba 
  mais!


Re: [obm-l] geometria plana

2003-09-14 Por tôpico Igor Castro



Se não tiver erro, segue uma solução 
assim..
-seja P o ponto de encontro de AC e 
BE.
- Tri APE eh semelhante ao Tri ABC - AP/5 = 
AE/sqrt(34)
- Tri APE eh semelhante ao Tri BPC - AE/5 = 
AP/(sqrt(34) - AP)
Então vc tem duas equações e duas variáveis. O 
sistema eh seu : )
Espero ter ajudado

Igor Castro

  - Original Message - 
  From: 
  andre resende 
  To: [EMAIL PROTECTED] 
  Sent: Sunday, September 14, 2003 1:57 
  AM
  Subject: [obm-l] geometria plana
  
  Alguém me dá uma luz?
  
  Considere um retângulo ABCD e um ponto E do lado 
  AD. Determine o comprimento do segmento AE, sabendo que BE e AC são 
  perpendiculares e que AB = 3 e AD = 5.
  
  Obrigado,
  André 
Resende


Re: [obm-l] Idades

2003-09-14 Por tôpico Igor Castro
Sua idade: AB

A+B=10
BA= AB +72-  9B - 9A = 72 -  B - A = 8
- B=9 e A=1
Sua idade: 19
[]´s
Igor Castro

- Original Message -
From: [EMAIL PROTECTED]
To: [EMAIL PROTECTED]; [EMAIL PROTECTED]
Sent: Sunday, September 14, 2003 2:12 PM
Subject: [obm-l]


 OLÁ AMIGOS; ALGUMAS QUESTÕES DE IDADES;

 1)MINHA IDADE É UM NÚMERO DE 2 ALGARISMOS. SOMANDO-SE  ELES OBTEMOS 10
 INVERTENDO A POSIÇÃO DOS NÚMEROS OBTEMOS OUTRO NÚMERO 72 UNIDADES A MAIS.
 QUAL É A MINHA IDADE?

 _
 Voce quer um iGMail protegido contra vírus e spams?
 Clique aqui: http://www.igmailseguro.ig.com.br
 Ofertas imperdíveis! Link: http://www.americanas.com.br/ig/

 =
 Instruções para entrar na lista, sair da lista e usar a lista em
 http://www.mat.puc-rio.br/~nicolau/olimp/obm-l.html
 =



=
Instruções para entrar na lista, sair da lista e usar a lista em
http://www.mat.puc-rio.br/~nicolau/olimp/obm-l.html
=


Re: [obm-l] Treinamento no Rio

2003-03-08 Por tôpico Igor Castro
Está definido que a reunião ocorrera somente neste horario e nesse dia
mesmo? Por que creio que muitos alunos interessados, como eu, não poderão ir
frequentemente devido ao colégio e curso que muitos fazem. Mas também não
vejo um horario/dia que facilite a todos rapidamente. Acho que seria bem
legal mesmo que se formasse um grupo de estudo para as olímpiadas no
RJ(como tem no CE), por isso, acho que todos os esforços deveriam ser feitos
para atrair o maior número possível de pessoas(alunos e profs). Bem, é só
uma colocação do meu ponto de vista, espero que a idéia possa ser
amadurecida e que se encontre o melhor para todos.
Abraços,
Igor...
- Original Message -
From: Carlos Gustavo Tamm de Araujo Moreira [EMAIL PROTECTED]
To: [EMAIL PROTECTED]
Sent: Friday, March 07, 2003 1:25 PM
Subject: Re: [obm-l] Treinamento no Rio


Caros colegas,
Estou escrevendo para lembrar da reuniao de segunda...
Abracos,
 Gugu

 
Caros colegas,
Na primeira segunda-feira depois do carnaval (10/2), no IMPA, as 14:00
 horas comecam as reunioes semanais de treinamento olimpico abertas ao
 publico, que visam entre outras coisas treinar para a IMO. Somos
responsaveis
 por estas reunioes eu e o Luciano, mas deveremos tambem ter aulas de
outros
 ilustres colegas (oi Nicolau! oi Okakamo! oi Morgado! oi Wagner!). Estao
 todos convidados (especialmente o pessoal do Rio...)!
Abracos,
Gugu
 
 =
 Instruções para entrar na lista, sair da lista e usar a lista em
 http://www.mat.puc-rio.br/~nicolau/olimp/obm-l.html
 O administrador desta lista é [EMAIL PROTECTED]
 =

 =
 Instruções para entrar na lista, sair da lista e usar a lista em
 http://www.mat.puc-rio.br/~nicolau/olimp/obm-l.html
 O administrador desta lista é [EMAIL PROTECTED]
 =



=
Instruções para entrar na lista, sair da lista e usar a lista em
http://www.mat.puc-rio.br/~nicolau/olimp/obm-l.html
O administrador desta lista é [EMAIL PROTECTED]
=


[obm-l] Dúvida familiar

2003-02-11 Por tôpico Igor Castro



Caros companheiros da lista, 
ao conversar com um dos meus cunhados ele me 
perguntou, sabendo pelo meu interesse por matemática, como faria para achar a 
aceleração com que sobe o nível de água em um cone com a ponta 
paracima se começarmos a enche-lo de água 
em uma determinada vazão V e sendoraioinferior do cone R e altura H. 
Pensei bastante e não consegui desenvolver nada útil. O problema ele criou 
na hora, então queria saber:
Tem solução? Qual?
Caso positivo, os dados da Vazão, 
daalturae do Raio são suficientes?
Bem, agradeço desde já e espero que o assunto não 
esteja muito off-topic por se tratar mais de física do que 
matemática.


[obm-l] Estadual-RJ

2002-09-01 Por tôpico Igor Castro


Alguém tem a resposta do primeiro 
problema do nivel 3 da segunda fase da Estadual-RJ que teve na puc 
ontem???


[obm-l] Canal de IRC...

2002-08-26 Por tôpico Igor Castro



Bem, que rede e qual o nome do canal que vocês 
sugerem?
to dentro...
[]'s


[obm-l] Geo Plana..

2002-07-03 Por tôpico Igor Castro



Olá amigos.. alguem pode dar uma ajuda nesse 
problema de geometria que não está saindo? 


A medida do angulo "a" na figura, sendo AM a metade 
de MB, é: (segue figura em anexo)
attachment: geo.jpg

[obm-l] Re: [obm-l] Quadriláteros

2002-06-20 Por tôpico Igor Castro

Fala ae rick
Na primeira ache as duas diagonais(usando hiparco e depois ptolomeu), em
seguida aplique a relação da mediana de euller...
a para achar o raio, use a formula abc/4R em um triangulo que possua uma
diagonal como lado(assim ele estará inscrito)...
no segundo analisei apenas com existencia de triangulos e cheguei que o
perimetro é maior que 10... mas um valor exato n achei...
[]'s

- Original Message -
From: [EMAIL PROTECTED]
To: [EMAIL PROTECTED]
Sent: Wednesday, June 19, 2002 11:10 PM
Subject: [obm-l] Quadriláteros


 Olá amigos , será que poderiam me ajudar nestas duas questões :
 1- Seja um quadrilátero inscritível ABCD cujos lados AB , BC , CD e DA
medem
 respectivamente 1 , 2 , 2 e 3 .Calcule a mediana de Euller do quadrilátero
 e o raio do círculo circunscrito.


 2-Um quadrilátero convexo O tem diagonais respectivamente iguais a 4 e 6
 .Qual um possível valor para o seu perímetro .

 Abraço..


   
  |-=Rick-C.R.B.=- |
  |ICQ 124805654   |
  |e-mail [EMAIL PROTECTED]  |
   


 --
 Use o melhor sistema de busca da Internet
 Radar UOL - http://www.radaruol.com.br



 =
 Instruções para entrar na lista, sair da lista e usar a lista em
 http://www.mat.puc-rio.br/~nicolau/olimp/obm-l.html
 O administrador desta lista é [EMAIL PROTECTED]
 =



=
Instruções para entrar na lista, sair da lista e usar a lista em
http://www.mat.puc-rio.br/~nicolau/olimp/obm-l.html
O administrador desta lista é [EMAIL PROTECTED]
=



[obm-l] Limites?!?!

2002-05-31 Por tôpico Igor Castro



Olá colegas da lista, 
estou iniciando ainda neste assunto mas alguém 
poderiadar uma ajuda neste limite? 

LIM 
[sqrt(x+2) + sqrt(x)] / x
x- -1

não consigo fugir da indeterminação ou de uma 
resposta com "i"(é valido para respostas de limite?)
ou talvez o limite nem exista... deixo a analise 
para vcs.. : )
agradeço desde já...
[]'s


[obm-l] Duvidas, L.G. por favor...

2002-04-24 Por tôpico Igor Castro



Estava dando uma lida sobre lugares geométricos e 
tive umas aulas, mas sempre relacionado a geo analítica, até aí nenhum grande 
problema... o problema foi ao ver um problema de LG num livro de geo 
plana,não consegui formular uma resposta utilizando somente a geometria 
plana, nem ao menos prova-la efetivamente( provar que este seria o unico 
conjunto de pontos que satisfaz a propriedade) bem, aqui vão os problemas, se 
alguém puder ajudar...

1) Determine o lugar geométrico dos pontos P tais 
que PA^2 + 3PB^2 = K^2 , K constante...
ps: PA e PB são segmentos...


2) Determinar o lugar geométrico dos pontos cuja 
diferença dos quadrados das distâncias a dois pontos fixos A e B é constante e 
igual a K^2.

Agradeço desde já...
[]'s



[obm-l] Problema de Função...

2002-04-13 Por tôpico Igor Castro



Olá colegas da lista,
gostaria de ajuda neste problema por que na 
minha resolução acho sempre 64... mas não está nas opções do problema, creio que 
seja facil. Lá vai:
Seja A um conjunto de numeros reais tais que para 
toda m pertencente a A a função: (m/2 -2).x^2 -mx + 8, terá sempre duas 
raizes reais x1 e x2 satisfazendo
0 x1 = x2  5 . Tomando K como o menor 
numero inteiro pertencente a A e fazendo na função m=K, as raizes obtidas x1 e 
x2 tem por soma de seus quadrados:
a) 20 b)25 
c)30 d)41 e)49

agradeço desde já...
[]'s
ps: = significa maior ou 
igual...


Re: Continuação de ...(ola amigos...)

2002-01-17 Por tôpico Igor Castro

Olá colegas da lista...
aqui envio algumas soluções para os dois problemas...
Primeira:
obs: essa questão se não me engano é do colégio naval e como ele diz um
valor possivel, lembro-me que dentre os valores das opções só havia um
possivel(12,5).. aki segue uma solução para este:

Trace uma diagonal(BD) e trace o seguemento pedido(chamemos MN, M e N ponto
médio dos lados), agora marque o ponto médio da diagona(chame de O) e
ligue-o a cada ponta do seguemento MN, note que um triangulo é formado,
pois como os lados opostos NÃO são paralelos, os dois seguementos OM e ON
não estarão na mesma reta, temos assim um triângulo de lados OM ON e MN, mas
OM é um segmento paralelo e forma um triangulo(MOD) semelhante de razão 1/2
ao triangulo DAB, portanto OM vale metade de AB=12, faça o analogo para ON e
temos OM=6 ON=8, e para o ultimo lado do triangulo OMN (MN) temos a relação
de existência para um triângulo:  8- 6  MN 8+6 ou seja,2MN14,
unica opção válida se não me engano era 12,5.. segue o desenho anexo..

Segunda:

1 naval é o angulo inscrito que corresponde ao arco r, já 2 navais é o
ângulo do vértice(apoiado no centro da circunferencia) que corresponde a
r... mas sabemos que 360 corresponde a 2.Pi.r .: 180 corresponde a Pi.r e
dois navais a r...
  então 180/pi corresponde a dois navais, 180(some dos angulos internos de
um triangulo) corresponde a 2navais.Pi
perdoem-me algum erro(principalmente os de português :P), espero poder ter
ajudado...
abraço a todos..
[]'s
- Original Message -
From: [EMAIL PROTECTED]
To: [EMAIL PROTECTED]
Sent: Thursday, January 17, 2002 11:26 PM
Subject: Continuação de(Olá amigos da lista , trago alguns exercícios bons.)




o outro é um problema de um triângulo inscrito em uma circunferência , mais
que esta muito confuso , vou coloca-lo exatamente como esta aqui no livro
, porque não tive praticamente nenhuma idéia , só consegui desenhar +/-
e enxergar algumas coisinhas grato.. : )
1) Seja ABCD um quadrilátero qualquer onde os lados opostos NÃO são
paralelos
. Se as medidas dos lados opostos AB e DC são, respectivamente ,igual a
12 e 16 , um valor possível para o segmento de extremo M ( ponto médio do
lado AD ) e N ( ponto médio do lado BC ) é:

2)Suponha que 1 (um ) naval (símbolo n )seja a medida de um ângulo convexo
, menor que um ângulo reto , inscrito em um círculo de raio r , cujos lados
determinam , nesse círculo , um arco de comprimento r . Assim sendo , a
soma das medidas dos ângulos internos de um triângulo é igual a :



--
Use o melhor sistema de busca da Internet
Radar UOL - http://www.radaruol.com.br








Re: Livro de Aritmética

2002-01-03 Por tôpico Igor Castro



Olá bernardo...
Olá colegas da lista...
Eu possuo esse livro sim, mas também é uma 
xerox, confesso que nunca vi a versão original de nenhum dos 3 
livros(aritmética, algebra e geometria), mas se lhe servir a xerox... entre em 
contato por e-mail o meu é: [EMAIL PROTECTED]
até+..
[]'s

  - Original Message - 
  From: 
  Bernardo 
  Salamon 
  To: [EMAIL PROTECTED] 
  Sent: Thursday, January 03, 2002 9:57 
  PM
  Subject: Livro de Aritmética
  
  

  

  

  
   
  
  Saudações a todos.
  
  Sou do Rio de Janeiro e gostaria de adquirir ou 
  xerocar um livro aspas antigo aspas de Aritmética do Comandante Paulo Pessoa 
  do qual fui aluno e hoje gostaria que meus filhos pudessem estudar por ele 
  também. Estou procurando este livro a mais de um ano. Quem tiver ou souber de 
  alguém que o tenha por favor me indiquem.
  
  Atenciosamente, 
Bernardo


Re: Onde compo o Saraeva?

2001-10-14 Por tôpico Igor Castro



Realmente vai ser muito dificil você achar ele para 
vender, mesmo em sebos, acho que a melhor forma é você tentar conseguir com 
algum professor, assim eu consegui dois originais... em ultimo caso você pode 
xerocar de alguém, porque realmente vale a pena...
[]'s

  - Original Message - 
  From: 
  Gustavo Nunes 
  Martins 
  To: [EMAIL PROTECTED] 
  Sent: Saturday, October 13, 2001 5:02 
  PM
  Subject: Onde compo o Saraeva?
  
  

  

  

  
   
  
  Dizem que ha um livro de fisica chamado Saraeva e que ele e tem 
  questoes
  muito interessantes. O titulo dele e Saraeva mesmo ou esse e o nome 
  do
  autor? Eu procurei esse livro na pagina de uma editora que parece 
  ter
  pego os livros da MIR (www.urss.ru), mas 
  nao achei nada (nao sei de qual
  editora esse livro e).
  
  Onde eu acho esse livro? Ele e tao bom assim?
  
  
  Obrigado,
  Gustavo
  
  


Re: Respostas CN

2001-10-03 Por tôpico Igor Castro



Gostaria de te indicar o PROBLEMAS SELECIONADOS DE 
MATEMÁTICA - Antônio Luis Santos e Raul F. W. Agostinho, mas vai ser dificil 
você achar... Esse foi o que eu mais usei para passare comparando com 
morgado e etc, este é muito melhor e ainda...rs... todo ano cai uma questão 
desse livro, desde 99... nem sei se podia dizer isto...
Igor..
[]'s


Re: Questao 5 do nivel 2=questao 4 do nivel 3

2001-09-14 Por tôpico Igor Castro



Não concordo...
é claro que não podemosexigir nada em uma 
competição sem qualquer fim, a não ser o aprendizado, como a obm, mas acho q é 
no mínimo muito importante que a banca crie questões para que não aconteça casos 
como esses de perder-se o tempo e não ter solução, e quando há a decisão mais 
justa acho que seria a anulação pois senão parece que esta brincando com o 
aluno, eh como vc dar um jogo de quebra cabeças que não tem como ser montado e 
pedir para que alguém descubra que naum pode ser montado e explique, a maioria n 
pensaria nisso mas sim que há uma solução e se naum esta conseguindo a culpa e 
dela e nunca da banca, isso talvez faça a pessoa ficar mto tempo um uma questão 
por culpa somente da prova, por que a principio ela está sempre 
correta...
- Original Message - 

  From: 
  Eduardo 
  Casagrande Stabel 
  To: [EMAIL PROTECTED] 
  Sent: Saturday, September 15, 2001 2:04 
  AM
  Subject: Re: Questao 5 do nivel 2=questao 
  4 do nivel 3
  
  

  

  

  
   
  
  Ola!
  
  Depois de reler o meu e-mail, eu percebi que ele soou muito mal. Eu nao 
  quis
  dizer que voce nao pode se sentir prejudicada, é claro que pode. O que 
  eu
  quis dizer foi seguinte: o fato de alguem, por exemplo, ter perdido o 
  tempo
  INTEIRO da prova para resolver todos os casos daquela questao é 
  irrelevante
  para a decisão que a banca deve tomar com relação a pontuação. Mesmo 
  a
  questão sendo muito difícil, impossível, errada ou extensa cabia a você 
  ter
  administrado bem o seu tempo para fazer as outras questões.
  
  Eduardo Casagrande Stabel.
  
  
  From: Fernanda Medeiros  
  lt;[EMAIL PROTECTED]
  
  É a minha opinião,senti-me prejudicada,posso? Ou não?
  
  From: "Eduardo Casagrande Stabel"  lt;[EMAIL PROTECTED]
  Reply-To: [EMAIL PROTECTED]
  To:  
  lt;[EMAIL PROTECTED]
  Subject: Re: Questao 5 do nivel 2=questao 4 do nivel 3
  Date: Fri, 14 Sep 2001 20:59:29 -0300
  
  Esse tipo de reclamaccao "eu fui prejudicado nas outras questoes por 
  que
  perdi muito tempo em uma" eh completamente absurda! Cabe a cada 
  aluno
  administrar seu tempo na hora da prova, e a comissao da obm nada tem 
  a ver
  com isso.
  
  Eduardo Casagrande Stabel.
  
  
  
  
  


Re: Maio

2001-09-01 Por tôpico Igor Castro



Eu fiz o problema dois a uns 4 dias tb.. e tb achei 
apenas o par 370-371..


Re: Urgente(ajuda algebra)

2001-04-17 Por tôpico Igor Castro



Caros colegas, 
tentando resolver este problema verifiquei 
que:
(a+b+c)^2 = a^2 + b^2 + c^2 + 2(ab + bc + ac) = 9 = 
13 + 2(ab + bc + ac) então ab + bc + ac = -2 e 

(a+b+c)^3 = a^3 + b^3 + c^3 + 3( a.b^2 + a^2.b + 
b^2.c + a^2.c + c^2.a + c^2.b + 2abc) = 
 27 = 27 + 3( a.b^2 + a^2.b + b^2.c + 
a^2.c + c^2.a + c^2.b + 2abc) então
a.b^2 + a^2.b + b^2.c + a^2.c + c^2.a + c^2.b + 
2abc = 0
fatorando a(ab+bc+ac) + b(ab+bc+ac) + c^2(a + b) = 
0 substituindo
-2a -2b + c^2(a + b) = 0 então (c^2 -2)(a + 
b) = 0
então ou a+b=0 ou c^2 - 2 =0, mas poderiamos ter 
fatorado de forma que "a" ou "b" ficassem separados como no caso do 
"c", encontrando c^2 =2, b^2=2 ou a^2 =2 mas se fossem verdadeiras a^2 + b^2 + 
c^2 = 6, contradizendo o problema, então pensei que a unica "saida" seria o 
fator (a+b) ser igual a 0, o que nos traria( por a+b+c=3) que c=3(confirmado em 
a^3 + b^3 + c^3 = 27) e que a = -b. Então teriamos a^2 + b^2 + c^2 = 2a^2 + c^2 
= 13 que a^2 = 2 e como ele pede a^4 + b^4 + c^4 teriamos
4 + 4 + 81 = 89, contrariando as outras respostas 
que foram enviadas, sei que há algum erro em meu raciocinio porque nada 

"diferencia" a, b e c. Assim sendo não poderiam ter 
valores diferentes(certo?). E gostaria de saber qual seria o procedimento 
correto após encontrar (a+b)(c^2 - 2)=0 visto que poderiamos também ter(pela 
mesma equação anterior) (a+c)(b^2 - 2)=0 ou
(c+b)(a^2 - 2)=0 ??? bem, agradeço desde 
já e espero não ter dito muita "besteira". obrigado.


  - Original Message - 
  From: 
  [EMAIL PROTECTED] 
  To: [EMAIL PROTECTED] 
  Sent: Saturday, April 14, 2001 9:02 
  PM
  Subject: Ajuda!!!Algebra
  
  
  Pessoal
  Sendo a+b+c=3 , a²+b²+c²=13 e a³+b³+c³=27 Como determino a elevado a 
  quarta
  potência + b elevado a quarta potência + c elevado a quarta 
  potência?
  
  Obrigado desde já
  
  
  
  ___
  
  http://www.zipmail.com.br O 
  e-mail que vai aonde você está.
  
  
  
  


Complicado...

2001-04-15 Por tôpico Igor Castro





Caros colegas, 
gostaria de agradecer ao professor carlos victor 
pela ajuda... e gostaria de por um problema que vem me intrigando à muito tempo, 
não consigo sair do "0" nele... agradeço desde já a ajuda...
SendoA_{n+1} = x^{A_n}para n = 0, 1, 
2,...e A_0 = 25 log 50 (base 5). Determinetodo x realque 
satisfaça a equação A_3 = 5.
tinha visto que:
A_3= x^(x^(x^25log50(base 5))) = 5 tah 
certo?


Ajuda...

2001-04-12 Por tôpico Igor Castro



Caros colegas, gostaria de ajuda nos seguintes 
problemas, estou estudando indução e estou com algumas dúvidas:
Demonstrar por indução:
1º 1^3 + 2^3 + 3^3 + ... + n^3 = 
[(n(n+1))/2]^2 .
2º n  2^n
3º Demonstrar que, traçando-se n retas em um 
plano, não se pode dividi-lo em mais de 2^n "partes".


Ajuda

2001-03-20 Por tôpico Igor Castro



Caríssimos colegas, estou precisando de ajuda nos 
seguintes problemas: (parecem simples)


1) Determine n natural para que 2^n + 1 seja 
divisível por 3. (resolver algebricamente)

2) Se x pertence à {0,1,2,...,25), para quantos 
valores de x, x^2 +3x +2 é múltiplo de 6?

Estava resolvendo esses problemas num capítulo de 
divisibilidade e congruências, se puderem usar só o 
conceito básicos dessas teorias, 
agradeço.

Igor Castro



Livros

2001-03-16 Por tôpico Igor Castro



Alguém sabe algum livro que tenha a teoria de 
congruencia, divisiblidade e etc, e que seja bom para me 
indicar?


Outra duvida...

2000-12-19 Por tôpico Igor Castro



"Sja n=1 um inteiro. Temos n lâmpadas alinhadas 
e numeradas, da esquerda para direita, de 1 a n. Cada lâmpada pode estar acesa 
ou apagada. a cada segundo, determina-se a lampada apagada de maior numero e 
inverte-se o estado desta(de acesa para apagada ou de apagada para acesa) e das 
lampadas posteriores(as lampadas de maior numero).
a) mostre que em algum momento todas as lampadas 
estarão acesas(e o processo se encerrará)"
chamei de 0 uma lampada apagada e 1 uma lampada 
acesa, portanto teriamos uma sequencia da seguinte forma: ...1010110111... ou 
...11011101110. No segundo caso no primeiro segundo a ultima lampada sera 
invertida(somente ela, pois não há nenhuma posterior) entào no proximo segundo a 
lampada apagada logo anterior inverterá e todas as posteriores(todas acesas) 
inverterão tb, então teremos ...110 então no segundo seguinte a ultima 
inverterá(somente ela) e teremos ...1100001 e no proximo segundo a penultima 
lampada(ultima apagada) inverterá e a ultima tb entào teremos ...1100010 e 
no proximo segundo teremos denovo ...110 o q jah aconteceu e este 
processo se repetirá infinitamente tanto para o primeiro caso tanto para o 
segundo...
Estaria certo isso? anda não localizei o erro, 
gostaria de uma ajuda..obrigado..


será q ninguém me ajuda?

2000-12-19 Por tôpico Igor Castro



deixei duas duvidas aki na lista so obtive uma 
resposta, se puderem me ajudar ae eu agradeço... mesmo q seja mto facil pra 
vcs.. bem em todo caso vo copiar as duvidas
Primeira:

No seguinte problema:"Mostre que, pelo menos 30% 
dos naturais n entre 1 e 1.000.000,o primeiro digito de 2^n é 1." Estou 
com duvida em minha resolução, até porque não encontreierros em meu 
raciocinio, mas sei que há porque a "prova" naum está dando certa, gostaria q 
alguém desse uma olhada e me indicasse o erro, lah vai
sendo 2^0=1(n=0 não serve), 2^7=128, 2^10=1024 
temos2 ocorrencias do primeiro algarismo sendo 1 para os 10 primeiro 
valores de n, jah na sequencia dos 10 proximos temos 3 ocorrencias: 2^14=1..., 
2^17=1... e 2^20=1 e na sequencia seguinte também temos 3 
ocorrencias(2^24=1... 2^27=1.. e 2^30) ou seja, é uma especie de periodo que a 
cada 10 numeros( começando por 1) temos 3 ocorrencias, então o numero de 
ocorrencias até 1.000.000 é (1.000.000/10)*3 ou seja, o numero de periodos vezes 
3, pois em cada periodo temos 3 ocorrencias, o resultado é 300.000 mas devemos 
diminuir 2 pois a primeira ocorrencia(n=0) não serve e a ultima também não(n = 
1.000.000) pois ele diz valores de n ENTRE 1 e 1.000.000, então o numero de 
ocorrencias seria 300.000 - 2 = 299.998, mas o problema é q este valor é menor 
que 30% do numero de naturais entre 1 e 1.000.000 q é 299.999,4. 
Alguém pode me ajudar? desculpem-me se errei mto 
feio...

Segunda:

"Sja n=1 um inteiro. Temos n lâmpadas alinhadas 
e numeradas, da esquerda para direita, de 1 a n. Cada lâmpada pode estar acesa 
ou apagada. a cada segundo, determina-se a lampada apagada de maior numero e 
inverte-se o estado desta(de acesa para apagada ou de apagada para acesa) e das 
lampadas posteriores(as lampadas de maior numero).
a) mostre que em algum momento todas as lampadas 
estarão acesas(e o processo se encerrará)"
chamei de 0 uma lampada apagada e 1 uma lampada 
acesa, portanto teriamos uma sequencia da seguinte forma: ...1010110111... ou 
...11011101110. No segundo caso no primeiro segundo a ultima lampada sera 
invertida(somente ela, pois não há nenhuma posterior) entào no proximo segundo a 
lampada apagada logo anterior inverterá e todas as posteriores(todas acesas) 
inverterão tb, então teremos ...110 então no segundo seguinte a ultima 
inverterá(somente ela) e teremos ...1100001 e no proximo segundo a penultima 
lampada(ultima apagada) inverterá e a ultima tb entào teremos ...1100010 e 
no proximo segundo teremos denovo ...110 o q jah aconteceu e este 
processo se repetirá infinitamente tanto para o primeiro caso tanto para o 
segundo...
Estaria certo isso? anda não localizei o erro, 
gostaria de uma ajuda..obrigado


Uma duvida...

2000-12-18 Por tôpico Igor Castro



No seguinte problema:"Mostre que, pelo menos 30% 
dos naturais n entre 1 e 1.000.000,o primeiro digito de 2^n é 1." Estou 
com duvida em minha resolução, até porque não encontreierros em meu 
raciocinio, mas sei que há porque a "prova" naum está dando certa, gostaria q 
alguém desse uma olhada e me indicasse o erro, lah vai
sendo 2^0=1(n=0 não serve), 2^7=128, 2^10=1024 
temos2 ocorrencias do primeiro algarismo sendo 1 para os 10 primeiro 
valores de n, jah na sequencia dos 10 proximos temos 3 ocorrencias: 2^14=1..., 
2^17=1... e 2^20=1 e na sequencia seguinte também temos 3 
ocorrencias(2^24=1... 2^27=1.. e 2^30) ou seja, é uma especie de periodo que a 
cada 10 numeros( começando por 1) temos 3 ocorrencias, então o numero de 
ocorrencias até 1.000.000 é (1.000.000/10)*3 ou seja, o numero de periodos vezes 
3, pois em cada periodo temos 3 ocorrencias, o resultado é 300.000 mas devemos 
diminuir 2 pois a primeira ocorrencia(n=0) não serve e a ultima também não(n = 
1.000.000) pois ele diz valores de n ENTRE 1 e 1.000.000, então o numero de 
ocorrencias seria 300.000 - 2 = 299.998, mas o problema é q este valor é menor 
que 30% do numero de naturais entre 1 e 1.000.000 q é 299.999,4. 
Alguém pode me ajudar? desculpem-me se errei mto 
feio...



Questão de Geometria Plana

2000-12-05 Por tôpico Igor Castro



Gostaria de saber a resolução de uma questão de 
geometria plana do livro Questões de Geometria Plana de Edgar de Alencar, a 
página é 80 o número é 78, fala sobre uma corda AB e traça-se duas tangentes(uma 
de cada ponto da corda) q se encontram em um ponto C depois pega-se um ponto do 
arco delimitado pela corda tal que a distancia desse ponto até uma das tangentes 
é 9 e até a outra é 4, ele pede a distancia desse ponto até a corda 
AB..


Re: Questão de Geometria Plana

2000-12-05 Por tôpico Igor Castro
Title: Re: Questão de Geometria Plana



Desculpe, mas não entendi porque eles são 
semelhantes, se puder explicar agradeço... obrigado...

  - Original Message - 
  From: 
  Eduardo Wagner 
  To: [EMAIL PROTECTED] 
  Sent: Thursday, December 07, 2000 1:58 
  AM
  Subject: Re: Questão de Geometria 
  Plana
  
  

  

  

  
   Sejam:PC a perpendicular a 
  tangente por A,PD a perpendicular a tangente por B,PE a perpendicular 
  a AB.Os triangulos PAC e PBE são semelhantes.Os triangulos PAE e PBD 
  sao semelhantes.Conclua que PE^2 = PC.PD = 6.--From: 
  "Igor Castro" [EMAIL PROTECTED]To: "obm lista" 
  [EMAIL PROTECTED]Subject: 
  Questão de Geometria PlanaDate: Tue, Dec 5, 2000, 5:13
  Gostaria de saber a resolução de 
uma questão de geometria plana do livro Questões de Geometria Plana de Edgar 
de Alencar, a página é 80 o número é 78, fala sobre uma corda AB e traça-se 
duas tangentes(uma de cada ponto da corda) q se encontram em um ponto C 
depois pega-se um ponto do arco delimitado pela corda tal que a distancia 
desse ponto até uma das tangentes é 9 e até a outra é 4, ele pede a 
distancia desse ponto até a corda 
AB..


Problemas Selecionados de Matemática

2000-11-18 Por tôpico Igor Castro




Alguém ae possui o livro problemas selecionados de 
matemática, eu tenho e gostaria de ver as resoluções de algumas questões, ah e 
tb se o autor antonio luiz santos esyiver lendo esta msg gostaria de saber se 
existe mesmo o vol2 deste maravilhoso livro. uma é assim:
EM UMA ILHA DESERTA HAVIA CINCO HOMENS E UM MACACO. 
DURANTE O DIA OS HOMENS COLHERAM COCOS E DEIXARAM A PARTILHA PARA O DIA 
SEGUINTE. DURANTE A NOITE, UM DOS HOMENS ACORDOU E RESOLVEU PEGAR A SUA PARTE. 
DIVIDIU A PILHA DO COCO EM CINCO PARTES IGUAIS, OBSERVOU QUE SOBRAVA UM COCO, 
DEU ESSE COCO PARA O MACACO, RETIROU E GUARDOU A SUA PARTE. MAIS TARDE, O 
SEGUNDO HOMEM ACORDOU E FEZ A MESMA COISA QUE O PRIMEIRO, DANDO TAMBÉM UM COCO 
PARA O MACACO. SUCESSIVAMENTE, CADA UM DOS TRES HOMENS RESTANTES FEZ O MESMO QUE 
OS OUTROS DOIS, ISTO É DIVIDINDO OS COCOS EXISTENTES EM CINCO PARTES IGUAIS, 
DANDO UM COCO PARA O MACACO E GUARDANDO A SUA PARTE. NO DIA SEGUINTE, OS CINCO 
HOMENS REPARTIRAM OS COCOS RESTANTES EM CINCO PARTES IGUAIS, OBSERVARAM QUE 
SOBROU UM COCO, DERAM-NO PARA O MACACO E CADA UM PEGOU UMA PARTE. SE N É O MENOR 
NÚMERO DE COCOS QUE A PILHA INICIAL PODIA TER ENTÃO A SOMA DOS SEUS ALGARISMOS 
VALE:
naum sei se é dificil, mas naum estou 
conseguindo..obrigado..